Трехлепестковая роза в декартовых координатах: Ошибка 404. Запрашиваемая страница не найдена

Содержание

Полярная роза



Совершенно верно, речь пойдёт о цветке с лепестками:

Задача 118

Построить линии, заданные уравнениями в полярных координатах

а) ,           б)

Существует два подхода к построению полярной розы. Сначала пойдём по накатанной колее, считая, что полярный радиус не может быть отрицательным:

Решение: а) Найдём область определения функции:

Неравенство решим графически. Согласно геометрическим преобразованиям графиков, если аргумент функции умножить на , то её график сожмётся к оси   в  раз. Смотрим на нашу синусоиду и отвечаем на вопрос: на каких промежутках она
не ниже оси абсцисс?

Неравенству  удовлетворяет бесконечно много отрезков, но нас интересуют только два, и область определения нашей функции: .

Возможно, некоторым читателям более лёгким покажется аналитический способ нахождения области определения, условно назову его «нарезка круглого пирога». Резать будем на равные части и, прежде всего, найдём границы первого куска. Рассуждаем
следующим образом: синус неотрицателен, когда  его аргумент находится в пределах от 0 до  рад. включительно. В нашем примере:  .

Разделив все части двойного неравенства на 2, получаем искомый промежуток:
Теперь начинаем последовательно «нарезать равные куски по 90 градусов» против часовой стрелки:

– найденный отрезок , понятно, входит в область определения;
– следующий интервал  – не входит;
– следующий отрезок  – входит;
– и, наконец, интервал  – не входит.

Прямо, как по ромашке – «любит, не любит, любит, не любит» =) С тем отличием, что тут не гадание.

…Да, прямо какая-то любовь по-китайски получается….

Итак,  и линия  представляет собой розу с двумя одинаковыми лепестками. Чертёж вполне допустимо выполнить схематически, однако крайне желательно правильно найти и отметить вершины лепестков. Им соответствуют середины отрезков  области определения, которые в данном примере имеют очевидные угловые координаты . При этом длины лепестков составляют:

И слева вы видите закономерный результат заботливого садовника.

Следует отметить, что длину лепестка легко сразу усмотреть из уравнения  – так как синус ограничен: , то максимальное значение «эр» заведомо не превзойдёт двух.

 

 

б) Построим линию, заданную уравнением . Очевидно, что длина лепестка этой розы тоже равна двум, но, прежде всего, нас интересует

область определения. Применим аналитический метод «нарезки»: синус неотрицателен, когда его аргумент находится в пределах от нуля до «пи» включительно, в данном случае: . Делим все части неравенства на 3 и получаем первый промежуток: . Теперь начинаем «нарезку пирога равными кусками» по  рад. (60 градусов):

– отрезок  войдёт в область определения;
– интервал  – не войдёт;
– отрезок  – войдёт;
– интервал  – не войдёт;
– отрезок  – войдёт;
– интервал  – нет, процесс успешно завершён на отметке 360 градусов.

Таким образом, область определения: .

Построение. Если в предыдущем пункте всё благополучно обошлось прямыми углами и углами в 45 градусов, то здесь придётся немного повозиться. Найдём

вершины лепестков. Их длина  была видна с самого начала задания, осталось вычислить угловые координаты, которые равны серединам отрезков области определения:
а

Обратите внимание, что между вершинами лепестков должны обязательно получиться равные промежутки, в данном случае 120 градусов.

Чертёж желательно разметить на 60-градусные секторы (отграничены зелёными линиями) и провести направления вершин лепестков (серые линии). Сами вершины удобно наметить с помощью циркуля – единожды отмерить по линейке расстояние в 2 единицы и нанести три засечки на прочерченных направлениях в 30, 150 и 270 градусов.

Понимаю, что занятие хлопотное, но если хотите всё оформить по уму, то придётся потратить время.

Сформулируем общую формулу: уравнение вида ,  – натуральное),  задаёт полярную -лепестковую розу, длина лепестка которой равна .

Например, уравнение  задаёт четырёхлистник с лепестком в 5 единиц, уравнение  – 5-лепестковую розу с длиной лепестка в 3 ед. и т.д.

О втором подходе я хотел вообще умолчать, однако не могу пройти мимо – уж слишком он распространён. Суть состоит в том, что полярная роза часто рассматривается в обобщённых полярных координатах, где полярный радиус может быть отрицательным. Вопрос области определения отпадает, но появляются другие приколы.

Во-первых, разберёмся, как строить точки с отрицательным значением «эр». Если , то нужно мысленно найти точку с таким же углом, но радиуса  и

отобразить её симметрично относительно полюса. Вернёмся к первой полярной розе  и рассмотрим интервал , на котором полярный радиус отрицателен (рисунок слева). Как, например, изобразить точку ? Мысленно находим точку  (левый верхний сектор) и отображаем её симметрично относительно полюса в точку . Таким образом, когда угол принимает значения из интервала , то прорисовывается ещё один лепесток в правом нижнем секторе (тот  же рисунок слева).

И, соответственно, когда угол проходит значения , то прорисовывается
лепесток в противоположном (левом верхнем) секторе (рисунок справа).

Интересно отметить, что при таком подходе роза с нечётным количеством лепестков, в частности, роза  сохраняет своё количество лепестков. А происходит это по одной простой причине: когда угол проходит пустующие секторы

(посмотрите на нижний чертёж предыдущей страницы!), то полярный радиус принимает отрицательные значения, и из этих пустых секторов точки отображаются напротив – ровнёхонько накладываясь на «легальные» лепестки.

Сформулируем правило розы для обобщенной системы координат: уравнение вида ,  – натуральное) задаёт полярную розу с длиной лепестка , при этом:
1) если — чётное, то роза имеет ровно  лепестков;
2) если — нечётное, то роза имеет ровно  лепестков.

Например, роза   имеет 8 лепестков, роза  – пять лепестков, роза  – 12 лепестков, роза  – 7 лепестков и т.д.

А почему закономерность столь необычна, я только что проиллюстрировал выше.

Какую систему выбрать, «классическую» или обобщённую? Зависит от вашего учебного плана. «По умолчанию» я бы особо не рекомендовал использовать обобщенные полярные координаты, ибо, зачем вам лишние вопросы со стороны преподавателя?

Похожая задача для самостоятельного решения:

Задача 119

Построить линии, заданные уравнением в полярных координатах
а) ,          б)

Сформулировать общее правило о количестве и длине лепестков полярной розы вида ,  – натуральное)

В образце приведено решение в «классической» полярной системе.

Повторим схему решения:

– Сначала находим область определения. При этом для лучшего понимания своих действий постарайтесь соотнести аналитический способ «нарезки» с графической интерпретацией. По материалам статьи геометрические преобразования графиков

выясните, как выглядят, и при необходимости начертите графики функций .

– Находим угловые координаты вершин лепестков – они расположены ровно посередине промежутков области определения.

– Выполняем чертёж. Пойдёт схематическая версия, однако желательно разметить найденные секторы и угловые направления вершин лепестков (в случае необходимости – с помощью транспортира). Вершины удобно засекать циркулем, предварительно установив раствор, равный длине лепестка.

Существуют более солидные и общие формулы окружности, полярной розы и желающие могут с ними ознакомиться в других источниках информации. Я лишь ограничился практически значимыми (с моей точки зрения) примерами.

И сейчас мы систематизируем алгоритм построения линий в полярной системе координат

, и что немаловажно, значительно ускорим решение:

4.6. Как построить линию в полярных координатах?

4.4. Уравнение линии в полярных координатах

| Оглавление |



Автор: Aлeксaндр Eмeлин


Плоские кривые — лемнискаты, циклоиды, гипоциклоиды, цепная линия, трохоида

ЛЕМНИСКАТЫ
Уравнение в полярных координатах:
r2 = a2cos2θ

Уравнение в прямоугольных координатах:
(x2 + y2)2 = a2(x2 — y2)

Угол между AB’ или A’B и осью x = 45o

Площадь одной петли = a2/2

ЦИКЛОИДА
Уравнения в параметрической форме:

Площадь одной дуги = 3πa2

Длина дуги одной арки = 8a

Это кривая, описываемая точкой Р на окружности радиусом а, которая катится вдоль оси х.

ГИПОЦИКЛОИДЫ С ЧЕТЫРЬМЯ ОСТРИЯМИ


Уравнение в прямоугольных координатах:
x2/3 + y2/3 = a2/3

Уравнения в параметрической форме:

Площадь, ограниченная кривой = 3πa2/8

Длина дуги целой кривой = 6a

Это кривая, описываемая точкой Р на окружности радиусом a/4, которая катится внутри окружности радиусом a.

КАРДИОИДА
Уравнение: r = a(1 + cosθ)

Площадь, ограниченная кривой = 3πa2/2

Длина дуги кривой = 8a

Это кривая, описываемая точкой Р на окружности радиусом a, которая катится снаружи окружности радиусом a. Эта кривая также является частным случаем улитки Паскаля.

ЦЕПНАЯ ЛИНИЯ
Уравнение:
y = a(ex/a + e-x/a)/2 = acosh(x/a)

Это кривая, по которой бы повисла цепь, подвешенная вертикально от точки А к В.

ТРЕХЛЕПЕСТКОВАЯ РОЗА
Уравнение: r = acos3θ

Уравнение r = acos3θ подобно кривой, полученной вращением против часовой стрелки по кривой 30o или π/6 радиан.

В общем, r = acosnθ или r = asinnθ имеет n лепестков если n является нечетным.

ЧЕТЫРЕХЛЕПЕСТКОВАЯ РОЗА
Уравнение: r = acos2θ

Уравнение r = asin2θ подобно кривой, полученной вращением против часовой стрелки по кривой 45o или π/4 радиан.

В общем r = acosnθ или r = asinnθ имеет 2n лепестков если n — четное.

ЭПИЦИКЛОИДА
Параметрические уравнения:

Это кривая, описываемая точкой Р на окружности радиуса b, когда она катится по внешней стороне окружности радиусом а. Кардиоида является частным случаем эпициклоиды.

ОБЩАЯ ГИПОЦИКЛОИДА
Параметрические уравнения:

Это кривая, описываемая точкой Р на окружности радиуса b, когда она катится по внешней стороне окружности радиусом а.

Если b = a/4, кривая является гипоциклоидой с четырьмя остриями.

ТРОХОИДА
Параметрические уравнения:

Это кривая, описываемая точкой Р на дистанции b от центра окружности с радиусом а, когда она катится по оси x.
Если b укороченной циклоидой.
Если b > a, кривая имеет форму, показанную на рис. 11-11 и называется троходой.
Если b = a, кривая есть циклоидой.

ТРАКТРИСА
Параметрические уравнения:

Это кривая, описываемая конечной точкой Р натянутой струны длиной PQ, когда другой конец Q перемещается вдоль оси х.

ВЕРЗЬЕРА (ВЕРЗИЕРА) АНЬЕЗИ (ИНОГДА ЛОКОН АНЬЕЗИ)
Уравнение в прямоугольных координатах: y = 8a3/(x2 + 4a2)

Параметрические уравнения:

В. На рисунке переменная линия OA пересекающая y = 2a и круг с радиусом a с центром (0,a) в A и B соотвественно. Любая точка P на «локоне» определяется построением линий, параллельных к осям x и y, и через B и A соответственно и определяющие точку пересечения P.

ДЕКАРТОВ ЛИСТ
Уравнение в прямоугольных координатах:
x3 + y3 = 3axy

Параметрические уравнения:

Площадь петли 3a2/2

Уравнение асимптоты: x + y + a = 0.

ЭВОЛЬВЕНТА ОКРУЖНОСТИ
Параметрические уравнения:

Эта кривая, описанная конечной точкой P струны, когда она разматывается с круга с радиусом a.

ЭВОЛЬВЕНТА ЭЛЛИПСА
Уравнение в прямоугольных координатах:
(ax)2/3 + (by)2/3 = (a2 — b2)2/3

Параметрические уравнения:

Эта кривая является огибающей нормалью к эллипсу x2/a2 + y2/b2 = 1.

ОВАЛЫ КАССИНИ
Полярное уравнение: r4 + a4 — 2a2r2cos2θ = b4.

Это кривая, описываемая такой точкой P, что произведение ее расстояния от двух фиксированных точек [ расстояние 2a в сторону] есть постоянной b2.

Кривая, как на фигурах внизу, когда b a соответственно.

Если b = a, кривая есть лемниската

УЛИТКА ПАСКАЛЯ
Полярное уравнение: r = b + acosθ

Пусть OQ будет линией, соединяющей центр O с любой точкой Q на окружности диаметром a проходящей через O. Тогда кривая есть фокусом всех точек P, таких, что PQ = b.

Кривая, показанная на рисунках внизу когда b > a или b

ЦИССОИДА ДИОКЛА
Уравнение в прямоугольных координатах: y2 = x3/(2a — x)

Параметрические уравнения:

Это кривая, описываемая такой точкой P, что расстояние OP = расстоянию RS. Используется в задаче удвоения куба, т.e. нахождения стороны куба, который имеет удвоенный объем заданного куба

СПИРАЛЬ АРХИМЕДА
Полярное уравнение: r = aθ

Моделирование функций в полярной системе координат и их связь с природой презентация, доклад, проект

Слайд 1
Текст слайда:

Реферат на тему: «Моделирование функций в полярной системе координат и их связь с природой.»

Составил ученик 8 б класса Ившин Сергей.

Руководители:
Ветошкина Наталья Владимировна
Ившина Татьяна Генадиевна


Слайд 2
Текст слайда:

Содержание.

Введение.
Цели.
Задачи.
Гипотеза.
Противоречие, проблема.
Предмет, объект исследования.
Понятие о полярной системы координат.
Исследование некоторых графиков функции и их связь с природой.


Слайд 3
Текст слайда:

Введение

Я выбрал эту тему потому, что тему полярных координат не рассматривается в школьном курсе, но часто графики, построенные в полярных координатах, встречаются в жизни и их очень интересно строить с помощью компьютерных технологий. Я решил исследовать некоторые известные линии, построенных в параметрической системе координат и их связь с природой.


Слайд 4
Текст слайда:

Цели.

1) подготовить наглядный материал для построения графиков функции;
2) способствовать развитию аналитических способностей и возможностей учащихся видеть прекрасное в такой точной науке как математика.


Слайд 5
Текст слайда:

Задачи.

Построить, с помощью компьютерных программ графики функций.
Расширить понятия систем координат.
Рассмотреть графики кривых известных математиков в полярной системе координат.
Рассмотреть связь природы с графиками полярных координат.


Слайд 6
Текст слайда:

Гипотеза.

Некоторые люди заинтересованы в изучении систем координат, но не многие знают, что кроме декартовой системы координат есть ещё и полярная система координат, цилиндрические, сферические, декартовые в пространстве.


Слайд 7
Текст слайда:

Противоречие, проблема.

Противоречие.
В школе мы изучаем только одну систему координат, а существуют ли другие системы координат? Можно ли программировать построение графиков.
Проблема.
Можно ли строить графики, используя компьютерные технологии?


Слайд 8
Текст слайда:

Предмет, объект.

Объект: Полярная система координат.
Предмет: Графики функции известных математиков.


Слайд 9
Текст слайда:

Понятие о полярной системы координат.

Точку М на плоскости в полярной системе координат можно определить парой чисел (R, Z), где R – расстояние от полюса 0 и Z – угол между осью и прямой, соединяющей полюс и данную точку ( угол изменяется в направлении против часовой стрелки от оси).

Z

O

R

М


Слайд 10
Текст слайда:

Исследование некоторых графиков функции и их связь с природой.

Спираль Архимеда:
Спиралью Архимеда называют кривую, задающуюся формулой:
R = a * Z,
где — a коэффициент пропорциональности, а R и Z — полярные координаты каждой из точек этой кривой.
Свойства этой спирали впервые были изучены Архимедом. Кривая имеет бесконечное число витков. Расстояние между двумя последовательными витками является постоянной величиной. Точек перегиба она не имеет.
Посмотреть график этой функции в программе QBASIC можно по этой ссылке. Например а = 3, fimin, fimax = 0, 50


Слайд 11
Текст слайда:

Связь спирали Архимеда с природой.

Рога некоторых рогатых животных закручены по спирали Архимеда, например у вилорога и болотного козла. Бутоны розы тоже напоминают спираль. Так же спираль Архимеда используют в винтах самолетов и кораблей. Можно увидеть спираль Архимеда и в бытовых предметах.
Даже вселенная имеет вид спирали Архимеда.


Слайд 12
Текст слайда:

Улитка Паскаля.

Улиткой Паскаля называют кривую, которая задаётся формулой:
R = 2a COS Z + b,
в которой a является коэффициентом пропорциональности, b — некоторым числом, а R и Z — полярными координатами каждой из точек этой кривой. Можно просмотреть улитку Паскаля по ссылке. Для примера можно взять значение а = 2, fimin, fimax = 0, 6,3


Слайд 13
Текст слайда:

Связь улитки Паскаля с природой.

Улитка Паскаля очень часто встречается в природе. Листы лилии, листики некоторых деревьев, очертание ягод вишни, персика, яблока – всё это напоминает улитку Паскаля.


Слайд 14
Текст слайда:

РОЗЫ.

Розы – это функции заданные в полярных координатах, имеющие вид
R = a SIN (b*Z), в которой a является коэффициентом пропорциональности, b — некоторым числом, а R и Z — полярными координатами каждой из точек этой кривой. Розы можно просмотреть по ссылкам:
1. Трёхлепестковая роза, где а = 2, fimin = 0, fimax = 6.3
2. Восьмилепестковая роза, где а = 2, fimin = 0,
fimax = 6.3
3. Четырёхлепестковая роза, где а = 2, fimin = 0,
fimax = 6.3


Слайд 15
Текст слайда:

Связь роз с природой.

Розы встречаются почти везде. Почти каждый цветок – это роза.


Слайд 16
Текст слайда:

Связь роз с пчелами.

Пчелы используют полярные координаты для обмена информацией об источниках пищи. Найдя новый источник пищи, пчела-разведчица возвращается в улей и исполняет танец, на языке которого рассказывает, где находится клумба. Причём всё это похоже на двулепестковую розу. Таким образом пчела-разведчица сообщает другим пчелам полярные координаты нового источника пищи.


Слайд 17
Текст слайда:

Строфоида.

Строфоиды – это функции заданные в полярных координатах, имеющие вид
R = -a * cos(2 * Z) / cos (Z), где a является коэффициентом пропорциональности, а R и Z — полярными координатами каждой из точек этой кривой. График этой функции можно посмотреть по этой ссылке, в которой a = 10, fimin = -1, fimax = 1.


Слайд 18
Текст слайда:

Связь строфоиды с природой

Строфоида похожа на простейшие организмы. Её силуэты напоминают рыбу. Строфоида выглядит как почка дерева, как воздушный шар.


Слайд 19
Текст слайда:

Некоторые графики функции, в программе QBASIC

Спираль Галилея
Следующие спирали пока не имеют названий, т. к. они являются моим экспериментом.
Номер 1
Номер 2
Номер 3
Номер 4
Номер 5
Номер 6


Слайд 20
Текст слайда:

Вывод.

В школе широко используются задания на построение и исследование графиков функций. Я предлагаю для изучения этих тем использовать компьютерную программу QBASIC.
Работа способствует развитию познавательных интересов, повышению информационной грамотности, фундаментальному математическому образованию.
Выполняя работу я познакомился с ещё с одной системой координат. Графики построенные в полярных координатах интереснее и более тесно связаны с природой. В ходе своего исследования я убедился, что математика связанна с природой. Я составил графики в программе QBASIC, чтобы показать ещё одну связь мира с природой.


Геометрические приложения определенного интеграла в полярной системе координат — КиберПедия

Навигация:

Главная Случайная страница Обратная связь ТОП Интересно знать Избранные

Топ:

Процедура выполнения команд. Рабочий цикл процессора: Функционирование процессора в основном состоит из по­вторяющихся рабочих циклов, каждый из которых соответствует…

Организация стока поверхностных вод: Наибольшее количество влаги на земном шаре испаряется с поверхности морей и океанов…

Теоретическая значимость работы: Описание теоретической значимости (ценности) результатов исследования должно присутствовать во введении…

Интересное:

Подходы к решению темы фильма: Существует три основных типа исторического фильма, имеющих между собой много общего…

Берегоукрепление оползневых склонов: На прибрежных склонах основной причиной развития оползневых процессов является подмыв водами рек естественных склонов…

Средства для ингаляционного наркоза: Наркоз наступает в результате вдыхания (ингаляции) средств, которое осуществляют или с помощью маски…

Дисциплины:

Автоматизация Антропология Археология Архитектура Аудит Биология Бухгалтерия Военная наука Генетика География Геология Демография Журналистика Зоология Иностранные языки Информатика Искусство История Кинематография Компьютеризация Кораблестроение Кулинария Культура Лексикология Лингвистика Литература Логика Маркетинг Математика Машиностроение Медицина Менеджмент Металлургия Метрология Механика Музыкология Науковедение Образование Охрана Труда Педагогика Политология Правоотношение Предпринимательство Приборостроение Программирование Производство Промышленность Психология Радиосвязь Религия Риторика Социология Спорт Стандартизация Статистика Строительство Теология Технологии Торговля Транспорт Фармакология Физика Физиология Философия Финансы Химия Хозяйство Черчение Экология Экономика Электроника Энергетика Юриспруденция

⇐ ПредыдущаяСтр 3 из 3

 

Пусть задана полярная система координат с полюсом и полярной осью (рис. 56). Тогда любая точка на плоскости имеет в этой системе две координаты: полярный радиус , равный длине отрезка , и полярный угол , на который нужно повернуть полярную ось до совмещения с точкой . Обычно договариваются, что . Однако это ограничение не является универсальным. Из рис. 56 видно, что

 

 

(79)

и

(80)

причём для определения угла следует учитывать знаки и . Формулы (79) и (80) устанавливают связь между полярными и декартовыми координатами точки для случая, когда полярная ось совпадает с осью .

 

2.4.1. Вычисление площади

Рассмотрим криволинейный сектор, ограниченный лучами и кривой (рис. 57). Вычислим его площадь.

Для этого выделим элементарный криволинейный сектор, расположенный между лучами с полярным углом и полярным углом . За бесконечно малый элемент площади примем площадь кругового сектора переменного радиуса с центральным углом . Тогда

.

Проинтегрируем полученное равенство при изменении от до и получим

 

(81)

Пример 4. 1.1.Вычислить площадь фигуры, ограниченной линией , заданной в полярных координатах.

Решение. Заданная в условии линия является трехлепестковой розой (рис. 58).

Так как , то решая неравенство , , найдем те значения , для которых она определена. Она определена для значений

Ограниченная ею фигура состоит из трех фигур («лепестков») с одинаковой площадью. Более того, каждый из трёх «лепестков» имеет ось симметрии, делящую фигуру на две равновеликие части. Так что достаточно вычислить площадь одной из половинок «лепестка» и умножить её на 6. Рассмотрим половину «лепестка», ограниченную лучами . Тогда согласно формуле (81) получим

 

Пример 4.1.2.Вычислить площадь фигуры, ограниченной линией , заданной в полярных координатах.

Решение. Заданная в условии линия является кардиоидой (рис. 59).

 

Фигура симметрична относительно луча (оси ). Будем вычислять площадь одной из половинок кардиоиды, заключенной между лучами , и умножать её на 2. Применяя формулу (81), получим

 

Пример 4.1.3.Вычислить площадь фигуры, ограниченной линиями , , заданными полярными координатами, и расположенной вне круга.

Решение. Заданная в условии кривая является двухлепестковой розой, а кривая – окружностью радиуса 1 с центром в начале координат (рис. 60). Т.к. , то решая неравенство , найдем те значения , для которых двухлепестковая роза определена:

 

Эти два отрезка определяют два лепестка розы. При этом луч делит лепесток первой четверти круга на две равные части. Так что двухлепестковая роза состоит из четырёх половинок «лепестка» одинаковой площади.

Найдём теперь координаты точки пересечения окружности и половины лепестка розы, ограниченной лучами . Для этого решим уравнение

.

Его решение . Таким образом, четвертая часть искомой фигуры заключена между лучами и .

На рис. 60 эта часть заштрихована. Площадь всей фигуры получим, если площадь заштрихованной части умножим на 4.

Площадь заштрихованной фигуры будем искать как разность между площадью фигуры, ограниченной линиями , , , и площадью сектора круга , заключённого между теми же лучами и .

Тогда согласно формуле (81) получим

 

Пример 4.1.4.Вычислить площадь фигуры, ограниченной линией , заданной в полярной системе координат.

Решение. На рис. 61 изображена заданная фигура.

Т.к. для всех значений , то согласно формуле (81) получим

 

Пример 4.1.5.Вычислить площадь фигуры, ограниченной лемнискатой, заданной в полярной системе координат уравнением .

Решение. Т.к. , то, решая неравенство ( ), найдем те значения , для которых заданная кривая определена. Она определена для значений

При этом в силу периодичности функции лемниската обладает симметрией относительно лучей (рис. 62).

 

 

 

Чтобы получить площадь заданной фигуры, достаточно найти площадь её части, заключённой между лучами и , и умножить полученный результат на 4. Тогда согласно формуле (81) получим

 

Пример 4.1.6.Вычислить площадь меньшей из фигур, ограниченных линиями, заданными в полярной системе координат уравнениями .

Решение. Линия (рис. 63)

очевидным образом является прямой (см. формулы (77) – связь полярных и декартовых координат). Линия представляет собой окружность с центром в точке с полярными координатами и радиусом 4. Заданная фигура ограничена двумя различными линиями: прямой и окружностью. Найдем точку их пересечения. Для этого решим уравнение

Его решением является .

В силу симметрии заданной фигуры будем вычислять площадь её половины, расположенной в первой четверти полярной системы координат и заштрихованной на рис. 63, и умножать полученный результат на 2. Заштрихованная фигура ограничена прямой с уравнением при изменении переменной от 0 до , а на участке – окружностью . Воспользуемся теперь формулой (81) и получим:

.

 

Пример 4.1.7.Вычислить площадь фигуры, ограниченной линией, заданной в полярной системе координат уравнением .

Решение. Заданная фигура изображена на рис. 64.

 

Функция определена для всех . Площадь вычислим по формуле (81). Тогда

.

2.4.2. Вычисление длины дуги

Пусть требуется найти длину дуги кривой , заключенной между лучами (рис. 57). Для вычисления бесконечно малого элемента длины дуги воспользуемся формулой (56). Используем теперь формулы (79), связывающие полярные координаты точки на рассматриваемой дуге с декартовыми координатами:

Тогда

и

Следовательно,

, (82)

а

. (83)

Пример 4.2.1.Найти длину дуги кардиоиды, заданной в полярной системе координат уравнением .

Решение. Заданная кривая изображена на рис. 65. Учтём, что кардиоида симметрична относительно полярной оси. Для того, чтобы вычислить длину всей кривой, достаточно вычислить длину её половины для и умножить результат на 2. Для нахождения бесконечно малого элемента длины дуги воспользуемся формулой (82). Найдём

,

а затем

 

 

Отметим, что для . Тогда искомая длина вычисляется так

.

 

Рекомендуемая литература

1. Натансон И.П. Краткий курс высшей математики./ И.П. Натансон. – СПб.: Лань, 2005.

2. Смирнова В.Б. Неопределенный интеграл: учебное пособие / В.Б. Смирнова,Л.Е. Морозова. – СПб.: СПбГАСУ, 2010.

3. Нумеров С.Н. Определённый интеграл. Методические указания к выполнению задания по курсу «Математика» для студентов всех специальностей ЛИСИ. – Л.: ЛИСИ, 1984.

4. Письменный Д.Т. Конспект лекций по высшей математике. Ч. 1. М.: Айрис-пресс, 2007.

Содержание

 

1.ОПРЕДЕЛЁННЫЙ ИНТЕГРАЛ И ЕГО СВОЙСТВА 3

1. 1.Задача о площади криволинейной трапеции 3

1.2.Определение определенного интеграла 5

1.3.Свойства определенного интеграла, выражаемые равенствами 7

1.4.Свойства определенного интеграла, выражаемые неравенствам 10

1.5.Теорема о среднем значении 11

1.6.Теорема Барроу 14

1.7.Формула Ньютона-Лейбница. Вычисление определенного интеграла 16

1.8.Интегрирование по частям в определенном интеграле 20

1.9.Замена переменной в определенном интеграле 23

1.10.Несобственные интегралы 27

2.ПРИЛОЖЕНИЯ ОПРЕДЕЛЁННОГО ИНТЕГРАЛА 36

2.1.Общий подход в приложениях определенного интеграла 36

2.2.Геометрические приложения определенного интеграла 36

2.2.1.Вычисление площадей 36

2.2.2.Вычисление объёма тела через площадь его сечения 54

2.2.3.Вычисление объёма тела вращения 55

2.2.4.Длина дуги плоской кривой 69

2. 2.5.Вычисление площади поверхности тела вращения 74

2.3.Механические приложения определенного интеграла 81

2.4.Геометрические приложения определенного интеграла в полярной системе координат 85

2.4.1. Вычисление площади 85

2.4.2. Вычисление длины дуги 100

Рекомендуемая литература 102

 

 

 

 

ОПРЕДЕЛЕННЫЙ ИНТЕГРАЛ

 

Составители: Лидия Евсеевна Морозова

Вера Борисовна Смирнова

 

Редактор

Корректор

Компьютерная верстка

 

Подписано к печати 00.00.2005. Формат 60×84 1/16. Бум. Офсетная. Усл. печ. л. . Уч.-изд. л. . Тираж 2000 экз. Заказ . «С» . Санкт-Петербургский государственный архитектурно-строительный университет. 190005, Санкт-Петербург, 2-я Красноармейская, 4. Отпечатано на ризографе. 190005, Санкт-Петербург, 2-ая Красноармейская, 5.

[1]Функция называется кусочно-непрерывной на промежутке , если она имеет на этом промежутке конечное число разрывов I рода.

[2] В интегралах, участвующих в формуле (28), переменной интегрирования является x.

⇐ Предыдущая123

Общие условия выбора системы дренажа: Система дренажа выбирается в зависимости от характера защищаемого…

Организация стока поверхностных вод: Наибольшее количество влаги на земном шаре испаряется с поверхности морей и океанов (88‰)…

Механическое удерживание земляных масс: Механическое удерживание земляных масс на склоне обеспечивают контрфорсными сооружениями различных конструкций…

Поперечные профили набережных и береговой полосы: На городских территориях берегоукрепление проектируют с учетом технических и экономических требований, но особое значение придают эстетическим…



Урок по геометрии Исследование полярных координат доклад, проект

Слайд 1
Текст слайда:

Исследование полярных координат

Балашова Елена Владимировна


Слайд 2
Текст слайда:

Цель урока:
Рассмотрение полярных координат и нахождение их взаимосвязи с прямоугольными координатами.
Задачи урока:
1. Выяснить, когда и в связи с какими потребностями появилось понятие полярные координаты на основе анализа школьных учебников математики, математической справочной литературы, литературы по истории математики.
2. Исследовать взаимосвязь полярных и прямоугольных координат
3. Исследовать виды полярных координат
4. Изучить построение графиков функции в полярной системе координат
5. Построить графики функции в полярной системе координат


Слайд 3
Текст слайда:

Введение.

Любая точка на плоскости может быть однозначно определена при помощи различных координатных систем, выбор которых определяется различными факторами.
Способ задания начальных условий для решения какой–либо конкретной технической задачи может определить выбор той или иной системы координат. Для удобства проведения вычислений часто предпочтительнее использовать системы координат, отличные от декартовой прямоугольной системы. Наглядность представления окончательного ответа иногда тоже сильно зависит от выбора системы координат.


Слайд 4
Текст слайда:

Определение полярных координат
Под системой координат на плоскости понимается способ, позволяющий численно описать положение точки плоскости. Одной из таких систем является полярная система координат.
Полярная система координат задается точкой О, называемой полюсом, лучом ОР, называемым полярной осью, и единичным вектором того же направления, что и луч ОР.
Возьмем на плоскости точку М, не совпадающую с О. Положение точки М определяется двумя числами: ее расстоянием r от полюса О и углом φ, образованным отрезком ОМ с полярной осью (отсчет углов ведется в направлении, противоположном движению часовой стрелки) (рис.1).

Рис. 1

Для получения всех точек плоскости достаточно полярный угол φ ограничить промежутком [0;2π), а полярный радиус r — [0;∞). В этом случае каждой точке плоскости (кроме О) соответствует единственная пара чисел r и φ.


Слайд 5
Текст слайда:

Связь прямоугольных координат с полярными.
Если на плоскости дана полярная система координат, то этим определена и некоторая прямоугольная система координат: за начало координат в этой прямоугольной системе берём начало полярной системы; полярную полуось объявляем положительной полуосью абсцисс. Таким образом определена ось абсцисс (вместе с её направлением ). Так как в определение полярной системы координат входит и направление положительного вращения плоскости, то мы можем определить ось ординат как ту ось, в которую перейдёт ось абсцисс при повороте её на угол в положительном направлении. Полученную таким образом прямоугольную систему координат будем называть системой определённой данной полярной системой ( рис.2).
•Каждой полярной системе координат соответствует вполне определённая прямоугольная система, и обратно. •


Слайд 6
Текст слайда:

Как же связаны между собою координаты x, y и ,r?
Если наряду с полярными координатами (r,φ) точки плоскости (например, точки М) ввести также ее прямоугольные координаты, как это было показано на рис. 2, то связь между ними выразится очевидными формулами, которые позволяют перейти от полярных координат точки M к прямоугольным.

Также можно сделать обратный переход, от прямоугольных координат к полярным, по формулам:


Слайд 7
Текст слайда:

Зная эти форму мы можем представить уравнения прямой, окружности, эллипса, гиперболы и параболы в полярных координатах.

Уравнение прямой: Уравнение окружности: Уравнение эллипса, гиперболы и параболы:

Различие между Эллипсом, гиперболой и параболой будут значения угла ,которые они принимают.


Слайд 8
Текст слайда:

В полярной системе координат существуют кривы, характерные только этой системе:
Кохлеоида, Строфоида, спираль Архимеда, логарифмическая спираль и др. Рассмотрим каждую из них.
Кохлеоида – трансцендентная кривая, уравнение которой в полярных координатах:

Кохлеоида имеет бесчисленное множество завитков, проходящих через полюс и касающихся полярной оси (рис. 7).

Кохлеоида

Рис.7


Слайд 9
Текст слайда:

Строфоида

Строфоида (от греч. στροφή — поворот) — алгебраическая кривая 3-порядка. Строится так (рис.9): даны точка О и прямая, находящаяся от точки О на расстоянии ОА = а. Вокруг точки О вращается луч, пересекающий прямую в переменной точке В. Строфоида — множество точек Мi, i = 1, 2, таких, что BМ1 = BМ2 = AB.
Уравнение строфоиды:

Из истории: Считается, что строфоида впервые была рассмотрена французским математиком Жилем Робервалем в 1645 году. Роберваль называл эту кривую — «птероида» (от греч. πτερον— крыло). Название «строфоида» было введено в 1849 году.


Слайд 10
Текст слайда:

Спираль Архимеда

Спираль Архимеда — плоская кривая, которую описывает точка, движущаяся равномерно-поступательно от центра 0 по равномерно-вращающемуся радиусу. Изобретение этой кривой приписывается Конону Самосскому, хотя ее основные свойства описал именно Архимед. Ему (Архимеду), в частности, было известно, что расстояние между двумя последовательными витками спирали является постоянной величиной и равно 2π (рис. 10).
Пусть а>0. Будем задавать углу всевозможные значения . Множество всех точек с полярными координатами и (т.е. множество всех точек с координатами , где пробегает все значения ), образует кривую, называемую спиралью Архимеда.
Уравнение кривой:

Рис.10


Слайд 11
Текст слайда:

Кардиоида

По мнению математиков, придумавших название кривой, она отдаленно напоминает форму сердца (в переводе с греческого kardieidos — сердцеобразная)
Кардиоида используется как линия для вычерчивания профилей, если требуется, чтобы скользящий по профилю стержень совершал гармонические колебания. При этом скорость поступательного движения стержня будет изменяться без скачков. Этим свойством она выгодно отличается от спирали Архимеда, у которой, благодаря постоянности скорости стержня, в конце каждого хода стержня происходят удары (скорость скачком меняет значение скорости с v на —v), что вызывает быстрое изнашивание механизма.
Уравнение:


Слайд 12
Текст слайда:

Логарифмическая спираль

Логарифмическая спираль или изогональная спираль — особый вид спирали, часто встречающийся в природе. Логарифмическая спираль была впервые описана Декартом и позже интенсивно исследована Бернулли, который называл её Spira mirabilis, «удивительная спираль».
Логарифмическая спираль обладает рядом интересных свойств:
•расстояния между последовательными витками образуют геометрическую прогрессию;
•последовательность длин радиусов, образующих одинаковые углы друг с другом, также составляет геометрическую прогрессию;
•образующиеся в процессе расширения секторы, отсекаемые такими радиусами, подобны друг другу.
Уравнение:


Слайд 13
Текст слайда:

Семейство роз Гранди

В XVIII в. итальянский геометр Гвидо Гранди (1671—1742) создал розы. Розы Гранди радуют нас правильными и плавными линиями, но их очертания не каприз природы — они предопределены специально подобранными математическими зависимостями. Семейство роз Гранди имеет свойство, которое в природе не сразу и заметишь: так как , то вся кривая расположена внутри круга единичного радиуса. В силу периодичности тригонометрических функций роза состоит из одинаковых лепестков, симметричных относительно наибольших радиусов, каждый из которых равен 1.
Наиболее красивые «цветы» получаются при k = 2 (четырехлепестковая роза) и при k = 3 (трехлепестковая роза) (рис.12).
Уравнение:

где k — положительная постоянная


Слайд 14
Текст слайда:

Построение графиков функции в полярной системе координат.

В полярной системе координат так же, как и в декартовой, по графику функции можно построить график функции .
Это построение сводится к простым геометрическим преобразованиям графика функции согласно перечисленным ниже свойствам.
Основные свойства графиков функции в полярной системы координат:
1. График функции симметричен графику функции относительно полюса.
2. График функции симметричен графику функции относительно полярной оси.
3. График функции , где m>0, — это растянутый или сжатый вдоль полярной оси в m раз график функции
4. График функции — это график, полученный из графика функции с помощью поворота последнего на угол .
5. График функции — это график функции параллельно перенесённый вдоль полярной оси на величину b.


Слайд 15
Текст слайда:

Итоги урока

Выяснили откуда появилось понятие полярная система (полярные координаты), что они представляют собой.
Исследовали как взаимосвязаны декартовая и полярная системы координат.
Изучили какие существуют виды полярных координат и их свойства.
На основе всего исследования построили некоторые из видов полярных кривых.
Гипотеза моего исследования подтвердилась, существует прямая взаимосвязь между декартовой и полярной системами координат.


СПЕЦИАЛЬНЫЕ ПЛОСКИЕ КРИВЫЕ — Мегаобучалка

ЛЕМНИСКАТЫ
Уравнение в полярных координатах:
r2 = a2cos2θ

Уравнение в прямоугольных координатах:
(x2 + y2)2 = a2(x2 — y2)

Угол между AB’ или A’B и осью x = 45o

Площадь одной петли = a2/2

ЦИКЛОИДА
Уравнения в параметрической форме:

Площадь одной дуги = 3πa2

Длина дуги одной арки = 8a

Это кривая, описываемая точкой Р на окружности радиусом а, которая катится вдоль оси х.

ГИПОЦИКЛОИДЫ С ЧЕТЫРЬМЯ ОСТРИЯМИ
Уравнение в прямоугольных координатах:
x2/3 + y2/3 = a2/3

Уравнения в параметрической форме:

Площадь, ограниченная кривой = 3πa2/8

Длина дуги целой кривой = 6a

Это кривая, описываемая точкой Р на окружности радиусом a/4, которая катится внутри окружности радиусом a.

КАРДИОИДА
Уравнение: r = a(1 + cosθ)

Площадь, ограниченная кривой = 3πa2/2

Длина дуги кривой = 8a

Это кривая, описываемая точкой Р на окружности радиусом a, которая катится снаружи окружности радиусом a. Эта кривая также является частным случаем улитки Паскаля.

ЦЕПНАЯ ЛИНИЯ
Уравнение:
y = a(ex/a + e-x/a)/2 = acosh(x/a)

Это кривая, по которой бы повисла цепь, подвешенная вертикально от точки А к В.

ТРЕХЛЕПЕСТКОВАЯ РОЗА
Уравнение: r = acos3θ

Уравнение r = acos3θ подобно кривой, полученной вращением против часовой стрелки по кривой 30o или π/6 радиан.



В общем, r = acosnθ или r = asinnθ имеет n лепестков если n является нечетным.

ЧЕТЫРЕХЛЕПЕСТКОВАЯ РОЗА
Уравнение: r = acos2θ

Уравнение r = asin2θ подобно кривой, полученной вращением против часовой стрелки по кривой 45o или π/4 радиан.

В общем r = acosnθ или r = asinnθ имеет 2n лепестков если n — четное.

ЭПИЦИКЛОИДА
Параметрические уравнения:

Это кривая, описываемая точкой Р на окружности радиуса b, когда она катится по внешней стороне окружности радиусом а. Кардиоида является частным случаем эпициклоиды.

ОБЩАЯ ГИПОЦИКЛОИДА
Параметрические уравнения:

Это кривая, описываемая точкой Р на окружности радиуса b, когда она катится по внешней стороне окружности радиусом а.

Если b = a/4, кривая является гипоциклоидой с четырьмя остриями.

ТРОХОИДА
Параметрические уравнения:

Это кривая, описываемая точкой Р на дистанции b от центра окружности с радиусом а, когда она катится по оси x.
Если b < a, кривая имеет форму, показанную на рис. 11-10 и называется укороченной циклоидой.
Если b > a, кривая имеет форму, показанную на рис. 11-11 и называется троходой.
Если b = a, кривая есть циклоидой.

ТРАКТРИСА
Параметрические уравнения:

Это кривая, описываемая конечной точкой Р натянутой струны длиной PQ, когда другой конец Q перемещается вдоль оси х.

ВЕРЗЬЕ?РА (ВЕРЗИЕ?РА) АНЬЕ?ЗИ (ИНОГДА ЛО?КОН АНЬЕ?ЗИ)
Уравнение в прямоугольных координатах: y = 8a3/(x2 + 4a2)

Параметрические уравнения:

В. На рисунке переменная линия OA пересекающая y = 2a и круг с радиусом a с центром (0,a) в A и B соотвественно. Любая точка P на «локоне» определяется построением линий, параллельных к осям x и y, и через B и A соответственно и определяющие точку пересечения P.

ДЕКАРТОВ ЛИСТ
Уравнение в прямоугольных координатах:
x3 + y3 = 3axy

Параметрические уравнения:

Площадь петли 3a2/2

Уравнение асимптоты: x + y + a = 0.

ЭВОЛЬВЕНТА ОКРУЖНОСТИ
Параметрические уравнения:

Эта кривая, описанная конечной точкой P струны, когда она разматывается с круга с радиусом a.

ЭВОЛЬВЕНТА ЭЛЛИПСА
Уравнение в прямоугольных координатах:
(ax)2/3 + (by)2/3 = (a2 — b2)2/3

Параметрические уравнения:

Эта кривая является огибающей нормалью к эллипсу x2/a2 + y2/b2 = 1.

ОВАЛЫ КАССИНИ
Полярное уравнение: r4 + a4 — 2a2r2cos2θ = b4.

Это кривая, описываемая такой точкой P, что произведение ее расстояния от двух фиксированных точек [ расстояние 2a в сторону] есть постоянной b2.

Кривая, как на фигурах внизу, когда b < a или b > a соответственно.

Если b = a, кривая есть лемниската

УЛИТКА ПАСКАЛЯ
Полярное уравнение: r = b + acosθ

Пусть OQ будет линией, соединяющей центр O с любой точкой Q на окружности диаметром a проходящей через O. Тогда кривая есть фокусом всех точек P, таких, что PQ = b.

Кривая, показанная на рисунках внизу когда b > a или b < a соответственно. Если b = a, кривая есть кардоидой.

ЦИССОИДА ДИОКЛА
Уравнение в прямоугольных координатах: y2 = x3/(2a — x)

Параметрические уравнения:

Это кривая, описываемая такой точкой P, что расстояние OP = расстоянию RS. Используется в задаче удвоения куба, т.e. нахождения стороны куба, который имеет удвоенный объем заданного куба

СПИРАЛЬ АРХИМЕДА
Полярное уравнение: r = aθ

 

 

Подсчет лепестков роз | plus.maths.org

Простота и симметрия кривых Rhodonea или rose очаровывали математиков с тех пор, как они были впервые названы итальянским математиком Гвидо Гранди в 1700-х годах. Мы были очарованы интересным узором, созданным путем подсчета количества лепестков этих кривых.

Рисунок розы

Кривые розы не рисуются с использованием координат, соответствующих горизонтальной и вертикальной осям (называемых 9).0003 Декартовы координаты ). Вместо этого кривые-розы рисуются с использованием полярных координат . Первая из полярных координат точки: расстояние между точкой и полюсом (полюс — это другое название начала координат, и это расстояние часто называют радиусом ). Вторая полярная координата (читается как «тета»): угол, образованный лучом (начинающимся в полюсе и заканчивающимся точкой) и полярной осью (по существу -осью). (Подробнее о том, как рисовать кривые с полярными координатами, вы можете прочитать в статье Полярная сила. )

Кривая розы имеет общее уравнение либо или где — ненулевое действительное число и целое число. Вы можете увидеть, что происходит с разными значениями в примере ниже. Когда вы увеличиваете значение от 1 до 20, возникает интересный паттерн: посмотрите, сможете ли вы его обнаружить.

Кривая розы, построенная для a = 1, и значения n в диапазоне от 1 до 20. только лепестки. Этот шаблон возникает, если есть любое целое число, в том числе если оно было отрицательным. Мы изучим математику, стоящую за этим интересным паттерном, и увидим, что это всегда имеет место для количества лепестков кривой розы.

Позвольте мне посчитать пути

Чтобы начать объяснение этой модели, мы должны взглянуть на уравнения, которые создают эти кривые. Допустим, мы рассматриваем. Используя наши знания о тригонометрических функциях, таких как косинус, мы знаем, что он должен находиться в диапазоне от -1 до 1 и, следовательно, должен находиться в диапазоне от до .

График y =cos(4 x ) (где r =1, n =4), построенный в декартовых координатах.


Размышляя о том, что это означает для кривой розы, нарисованной в полярных координатах, вершины лепестков — это те точки на кривых, которые наиболее удалены от полюса; те точки, где достигает или . Глядя на кривую розы, мы видим 8 мест, где достигает 1 или -1, соответствующих вершинам каждого из лепестков.

Кривая розы График y =cos(4 x ) (где r =1, n =4), построенный с использованием полярных координат. Вершины лепестков соответствуют вершинам и впадинам косинусного графика выше.


Чтобы посчитать количество лепестков на кривой розы, нам нужно посчитать количество вершин. Но какие значения создают пик? Или, другими словами, какие значения позволяют достичь или ? Функция достигает или когда кратно . Таким образом, пики кривой розы возникают, когда она равна или , то есть когда кратна .

Математически говоря, пики лепестков возникают, когда , где — целое число. Однако мы рассматриваем только значения между и . Это связано с тем, что угол, определяемый в нашей полярной системе координат, совпадает с углом, определяемым с помощью и , поэтому, как только он превышает его, мы перерисовываем нашу розу.

Значения где

     

— значения где

     

или

     

С некоторой простой перестановкой мы можем найти с точки зрения , и :

     

где . Таким образом, для каждого из значений между и (само себя не считаем, так как здесь кривая начинает повторяться) мы получаем вершину лепестка на кривой розы. Казалось бы, это означает, что на кривой розы есть лепестки!

 

Однако это, кажется, противоречит нашему первоначальному наблюдению, что когда нечетно, есть только лепестки, а не лепестки. Чтобы понять, почему это не противоречие, мы должны провести некоторые исследования.

Положительные пики

Мы можем перечислить полярные координаты для каждого пика кривой розы с помощью формулы . Взяв все значения от до , мы получим пики, когда

     

с указанием полярных координат пиков:

     

Мы видим, что в любое нечетное время первая координата этих пиков отрицательна: . Поскольку первая полярная координата была определена как расстояние между соответствующей точкой и полюсом, это не имеет смысла: расстояние не может быть отрицательным. Однако существует соглашение об отрицательном радиусе в полярной системе координат: точка с отрицательным радиусом — это точно такая же точка, как и точка с таким же положительным радиусом, но в противоположном направлении от полюса. То есть это та же точка, что и .

Точка (3, π/3) находится напротив точки (-3, π/3) от полюса. Мы можем переместить одну точку в другую, добавив π ко второй полярной координате. Таким образом, мы можем переписать (-3, π/3) как (3, 4π/3).

Таким образом, для нашего списка пиков кривой розы, для тех, у которых первая координата отрицательная, мы можем добавить ко второй координате и поменять знак , и точки, описываемые этими новыми координатами, будут по-прежнему эквивалентны:

   

Итак, мы можем переписать наш список пиков следующим образом:

     

Эвены эксклюзивные

После переписывания нашего списка пиков кривой розы у нас есть пики при для четных и при для нечетных.

Если четное, то нечетное число, если нечетное. Таким образом, переписанные координаты для нечетных не соответствуют координатам ни одной из точек для четных значений . Таким образом, мы получаем различные точки.

Вы можете увидеть восемь различных лепестков кривой розы, нарисованные ниже:

Нечетное перекрытие

Если мы попробуем случай, когда нечетное, мы получим нечто совсем другое. Когда мы перечисляем значения, которые создают пики, и преобразуем их так, чтобы все радиусы были положительными, мы не получаем точки расстояния. Поскольку нечетно для отрицательных радиусов, а также нечетно, то число четное. Однако это означает, что перезаписанные координаты для некоторого нечетного значения являются лишь одной из уже перечисленных нами координат для некоторого четного значения! Поэтому каждая точка дублируется один раз и ровно один раз. Таким образом, вместо лепестков мы получаем лепестки.

Вы можете видеть перекрывающиеся лепестки для кривой розы, нарисованные ниже:

Мы исследовали этот шаблон для кривых роз, описанных , но та же самая линия рассуждений объясняет шаблон для кривых, описанных .


Об этой статье

Джон Экхарт

Джон Экхарт — бывший учитель математики с отличием и высшим образованием, интересующийся аналитической геометрией. В настоящее время работает специалистом по образованию в Центре космических полетов им. Маршалла.


Фрэнк Ли

Фрэнк Ли учится в старшей школе Sylvania Northview High School (Толедо, Огайо), любит геометрию. По заданию своего учителя по предварительному исчислению придумать доказательство этой аккуратной закономерности, найденной в кривых розы, он нашел нечто столь же точное:

полярных координат — Найдите площадь лепестка розы.

спросил

Изменено 5 лет, 8 месяцев назад

Просмотрено 68 тысяч раз

$\begingroup$

Если лист розы описывается уравнением $r = \sin 3\theta$, найдите площадь одного лепестка. {\pi/3}{1 — \cos\pars{6\theta} \over 2}\,\dd\theta \\[3мм]&= {1 \более 4}\bracks{{\pi \более 3} — {\sin\pars{2\pi} \более 6}} =\color{#0000ff}{\large{\pi \более 12}} \end{выравнивание}

$\endgroup$

1

Твой ответ

Зарегистрируйтесь или войдите в систему

Зарегистрируйтесь с помощью Google

Зарегистрироваться через Facebook

Зарегистрируйтесь, используя адрес электронной почты и пароль

Опубликовать как гость

Электронная почта

Требуется, но не отображается

Опубликовать как гость

Электронная почта

Требуется, но не отображается

Нажимая «Опубликовать свой ответ», вы соглашаетесь с нашими условиями обслуживания, политикой конфиденциальности и политикой использования файлов cookie

графиков — алгебра и тригонометрия OpenStax

Цели обучения

В этом разделе вы будете:

  • Проверять полярные уравнения на симметрию.
  • Графики полярных уравнений по точкам.

Планеты движутся в космосе по эллиптическим периодическим орбитам вокруг Солнца, как показано на (Рисунок). Они находятся в постоянном движении, поэтому фиксация точного положения любой планеты действительна только на мгновение. Другими словами, мы можем исправить только 9 планету.0003 мгновенная позиция. Это одно из применений полярных координат, представленное как [латекс]\,\влево(г,\тета\вправо).\,[/латекс]
. Мы интерпретируем [латекс]\,г\,[/латекс] как расстояние от солнца и [латекс]\,\тета\,[/латекс] как угловой азимут планеты или ее направление от фиксированной точки на солнце. В этом разделе мы сосредоточимся на полярной системе и графиках, которые создаются непосредственно из полярных координат.

Рисунок 1. Планеты движутся по эллиптической траектории, вращаясь вокруг Солнца. (кредит: модификация работы NASA/JPL-Caltech) 9{2}\,[/latex]описывает взаимосвязь между[латексом]\,x\,[/latex]и[латексом]\,y\,[/latex]на декартовой сетке, полярное уравнение описывает связь между [латексом]\,r\,[/латексом] и [латексом]\,\тета \,[/латекс] на полярной сетке. Напомним, что пара координат [латекс]\,\влево(r,\тета \вправо)\,[/латекс] указывает, что мы двигаемся против часовой стрелки от полярной оси (положительные x -оси) на угол [латекс] \,\theta ,\,[/latex]и продолжить луч от полюса (начала)[латекс]\,r\,[/латекс]единиц в направлении [латекс]\,\тета .\,[/ латекс]Все точки, удовлетворяющие полярному уравнению, находятся на графике.

Симметрия — это свойство, которое помогает нам распознавать и строить график любого уравнения. Если уравнение имеет график, симметричный относительно оси, это означает, что если мы сложим график пополам по этой оси, часть графика с одной стороны совпадет с частью с другой стороны. Выполнив три теста, мы увидим, как применять свойства симметрии к полярным уравнениям. Далее мы будем использовать симметрию (в дополнение к построению ключевых точек, нулей и максимумов [латекса]\,r)\,[/латекса] для построения графика полярного уравнения.

В первом тесте мы рассматриваем симметрию относительно линии [латекс]\,\theta =\frac{\pi }{2}\,[/латекс]( y -оси). Мы заменяем [латекс]\,\left(r,\theta\right)\,[/latex] на [латекс]\,\left(-r,-\theta \right)\,[/latex], чтобы определить, новое уравнение эквивалентно исходному уравнению. Например, предположим, что нам дано уравнение [латекс]\,r=2\mathrm{sin}\,\theta ;[/latex]

[латекс]\begin{array}{ll}\,\,\, \,r=2\mathrm{sin}\,\theta\hfill & \hfill \\ -r=2\mathrm{sin}\left(-\theta\right)\begin{array}{cccc}& & & \end{массив}\hfill & \text{Заменить}\,\left(r,\theta\right)\,\text{с}\left(-r,-\theta\right).\hfill \\ — r=-2\mathrm{sin}\,\theta\hfill & \text{Идентификация:}\mathrm{sin}\left(-\theta\right)=-\mathrm{sin}\,\theta .\hfill \\ \,\,\,\,\,r=2\mathrm{sin}\,\theta \hfill & \text{Умножить обе части на}-1.\hfill \end{массив}[/latex]

Это уравнение проявляет симметрию относительно прямой [латекс]\,\тета =\фракция {\pi }{2}.[/латекс]

Во втором тесте мы рассматриваем симметрию относительно полярной оси ( [латекс]\,x[/латекс]-ось). Мы заменяем [латекс]\,\left(r,\theta \right)\,[/latex] на [латекс]\,\left(r,-\theta \right)\,[/latex]или [латекс] \,\left(-r,\pi -\theta \right)\,[/latex] для определения эквивалентности между тестируемым уравнением и исходным. Например, предположим, что нам дано уравнение[латекс]\,r=1-2\mathrm{cos}\,\theta .[/латекс]

[латекс]\begin{array}{ll}r=1-2\mathrm{cos}\,\theta \hfill & \hfill \\ r=1-2\mathrm{cos}\left(-\theta \right)\begin{array}{cccc}& & & \end{array}\hfill & \text{Replace}\left(r,\theta \right)\,\text{with}\,\left(r ,-\theta \right).\hfill \\ r=1-2\mathrm{cos}\,\theta \hfill & \text{четная/нечетная идентичность}\hfill \end{массив}[/latex]

График этого уравнения проявляет симметрию относительно полярной оси.

В третьем тесте мы рассматриваем симметрию относительно полюса (начала). Мы заменяем [латекс]\,\left(r,\theta \right)\,[/latex] на [латекс]\,\left(-r,\theta \right)\,[/latex], чтобы определить, проверенное уравнение эквивалентно исходному уравнению. Например, предположим, что нам дано уравнение [латекс]\,r=2\mathrm{sin}\left(3\theta \right).[/latex]

[латекс]\begin{array}{c}\,\,\,\,\,r=2\mathrm{sin}\left(3\theta\right)\\ -r=2\mathrm{sin }\left(3\theta \right)\end{array}[/latex]

Уравнение не прошло тест на симметрию, но это не означает, что оно несимметрично относительно полюса. Прохождение одного или нескольких тестов на симметрию подтверждает, что симметрия будет отображаться на графике. Однако непрохождение тестов на симметрию не обязательно означает, что график не будет симметричным относительно линии [латекс]\,\тета =\фракция{\пи} {2},\,[/латекс] полярной оси или столб. В этих случаях мы можем подтвердить существование симметрии, нанеся отражающие точки на кажущуюся ось симметрии или полюс. Проверка на симметрию — это метод, который упрощает построение графиков полярных уравнений, но его применение несовершенно.

Проверка симметрии

Полярное уравнение описывает кривую на полярной сетке. График полярного уравнения может быть оценен для трех типов симметрии, как показано на (Рисунок).

Рис. 2. (a) График симметричен относительно прямой [латекс]\,\theta =\frac{\pi }{2}\,[/латекс](ось Y) при замене[латекс ]\,\left(r,\theta \right)\,[/latex]с [латексом]\,\left(-r,-\theta \right)\,[/latex] дает эквивалентное уравнение. (b) Граф симметричен относительно полярной оси (оси x), если заменить [латекс]\,\left(r,\theta \right)\,[/latex]на [латекс]\,\left( r,-\theta\right)\,[/latex]или[latex]\,\left(-r,\mathrm{\pi -}\theta\right)\,[/latex] дает эквивалентное уравнение. (c) Граф симметричен относительно полюса (начала координат), если заменить [латекс]\,\left(r,\theta\right)\,[/latex] на [латекс]\,\left(-r, \theta \right)\,[/latex] дает эквивалентное уравнение.

How To

Имея полярное уравнение, проверьте его симметрию.

  1. Подставьте соответствующую комбинацию компонентов вместо [латекс]\,\left(r,\theta \right):[/latex][латекс]\,\left(-r,-\theta \right)\, [/latex]for[латекс]\,\theta =\frac{\pi }{2}\,[/latex]симметрия;[латекс]\,\left(r,-\theta \right)\,[/ латекс] для симметрии полярной оси; и [латекс]\,\влево(-r,\тета\вправо)\,[/латекс]для симметрии относительно полюса.
  2. Если результирующие уравнения эквивалентны в одном или нескольких тестах, график обеспечивает ожидаемую симметрию.

Проверка полярного уравнения на симметрию

Проверка уравнения[латекс]\,r=2\mathrm{sin}\,\theta \,[/латекс] на симметрию.

[reveal-answer q=»121887″]Показать решение[/reveal-answer]
[hidden-answer a=»121887″]

Тест на каждый из трех типов симметрии.

Example: -r = 2sin(theta). r=-2sin(theta) which does not equal 2sin(theta), so it fails the test.»>
1) Замена [латекс]\,\left(r,\theta \right)\,[/latex]на [латекс]\,\left(-r,-\theta \right)\,[ /latex] дает тот же результат. Таким образом, граф симметричен относительно прямой [латекс]\,\тета =\фракция{\пи }{2}.[/латекс] [латекс]\begin{array}{ll}-r=2\mathrm{sin}\left(-\theta\right)\hfill & \hfill \\ -r=-2\mathrm{sin}\,\ theta \hfill & \text{Четно-нечетная идентичность}\hfill \\ \,\,\,\,r=2\mathrm{sin}\,\theta \hfill & \text{Multiply}\,\text{by }\,-1\hfill \\ \text{пройдено}\hfill & \hfill \end{массив}[/latex]
2) Замена [латекс]\,\тета \,[/латекс] на [латекс]\,-\тета \,[/латекс] не дает того же уравнения. Следовательно, график не проходит тест и может быть или не быть симметричным относительно полярной оси. [латекс]\begin{array}{ll}r=2\mathrm{sin}\left(-\theta\right)\hfill & \hfill \\ r=-2\mathrm{sin}\,\theta \ hfill & \text{Четно-нечетная идентичность}\hfill \\ r=-2\mathrm{sin}\,\theta \ne 2\mathrm{sin}\,\theta \hfill & \hfill \\ \text{Ошибка }\hfill & \hfill \end{массив}[/latex]
3) Замена [латекс]\,r\,[/латекс] на [латекс]–r\,[/латекс] меняет уравнение и не проходит тест. Граф может быть или не быть симметричным относительно полюса. [латекс]\begin{array}{l}-r=2\mathrm{sin}\,\theta \hfill \\ \text{ }r=-2\mathrm{sin}\,\theta \ne 2\ mathrm{sin}\,\theta\hfill \\ \text{Failed}\hfill \end{array}[/latex]

[/hidden-answer]

Анализ

С помощью графического калькулятора мы можем увидеть, что уравнение[латекс]\,r=2\mathrm{sin}\,\theta \,[/latex] равно круг с центром в[латекс]\,\влево(0,1\вправо)\,[/латекс]с радиусом[латекс]\,r=1\,[/латекс] и действительно симметричен линии[латекс] \,\theta =\frac{\pi }{2}.\,[/latex]Мы также можем видеть, что график не симметричен относительно полярной оси или полюса. См. (Рисунок).

Рис. 3.

Попробуйте

Проверьте уравнение на симметрию: [латекс]\,r=-2\mathrm{cos}\,\theta .[/latex]

[reveal-answer q=»fs-id1165135664060″]Показать решение[/reveal- ответ]
[скрытый ответ a = «fs-id1165135664060»]

Уравнение не проходит тест на симметрию относительно линии[латекс]\,\тета =\фракция{\пи }{2}\,[/латекс]и относительно полюса. Он проходит тест на симметрию полярной оси.

[/hidden-answer]

Построение графика полярных уравнений по точкам

Для построения графика в прямоугольной системе координат мы строим таблицу [латекс]\,х\,[/латекс] и [латекс]\,у\ ,[/латекс]значения. Чтобы построить график в полярной системе координат, мы строим таблицу значений [латекс]\,\тета\,[/латекс] и [латекс]\,r\,[/латекс]. Мы вводим значения[латекс]\,\тета\,[/латекс] в полярное уравнение и вычисляем[латекс]\,р.\,[/латекс]Однако, используя свойства симметрии и находя ключевые значения[латекс ]\,\theta \,[/latex] и [latex]\,r\,[/latex] означают, что потребуется меньше вычислений.

Нахождение нулей и максимумов

Чтобы найти нули полярного уравнения, мы решаем значения [латекс]\,\тета\,[/латекс], что дает [латекс]\,r=0.\, [/latex] Напомним, что для нахождения нулей полиномиальных функций мы устанавливаем уравнение равным нулю, а затем решаем для[latex]\,x.\,[/latex]. Мы используем тот же процесс для полярных уравнений. Установите[latex]\,r=0,\,[/latex]и решите для[latex]\,\theta .[/latex]

Для многих форм, с которыми мы столкнемся, максимальное значение полярного уравнения равно можно найти, подставив те значения [латекс]\,\тета \,[/латекс] в уравнение, которые приводят к максимальному значению тригонометрических функций. Рассмотрим [латекс]\,r=5\mathrm{cos}\,\theta ;\,[/латекс]максимальное расстояние между кривой и полюсом составляет 5 единиц. Максимальное значение функции косинуса равно 1, когда [латекс]\,\тета = 0,\,[/латекс], поэтому наше полярное уравнение имеет вид [латекс]\,5\mathrm{cos}\,\тета,\,[ /latex] и значение[latex]\,\theta =0\,[/latex] даст максимальное значение[latex]\,|r|.[/latex]

Аналогично, максимальное значение функции синуса равно 1, когда [латекс]\,\тета =\фракция{\пи }{2},\,[/латекс] и если наше полярное уравнение равно [латекс]\,r =5\mathrm{sin}\,\theta ,\,[/latex]значение [латекс]\,\theta =\frac{\pi }{2}\,[/latex] даст максимум[латекс] \,|r|.\,[/latex] Мы можем найти дополнительную информацию, вычислив значения [latex]\,r\,[/latex], когда [latex]\,\theta =0. {- 1}0\hfill & \hfill \\ \,\,\,\,\,\ ,\,\,\,\,\,\theta =n\pi \hfill & \text{где }n\text{ — целое число}\hfill \end{массив}[/latex]

Подставьте любое из значений [latex]\,\theta \,[/latex] в уравнение. Мы будем использовать [латекс]\,0.[/латекс]

[латекс]\begin{array}{l}\begin{array}{l}\\ r=2\mathrm{sin}\left(0\ right)\end{array}\hfill \\ r=0\hfill \end{array}[/latex]

Точки[latex]\,\left(0,0\right)\,[/latex]и [latex]\,\left(0,±n\pi \right)\,[/latex] — нули уравнения. Все они совпадают, поэтому на графике видна только одна точка. Эта точка также является единственным пересечением полярной оси.

Чтобы найти максимальное значение уравнения, посмотрите на максимальное значение тригонометрической функции[латекс]\,\mathrm{sin}\,\theta ,\,[/латекс], которая возникает, когда[латекс]\,\тета =\frac{\pi }{2}±2k\pi \,[/latex], что приводит к [латексу]\,\mathrm{sin}\left(\frac{\pi }{2}\right)=1. \,[/latex]Замените [латекс]\,\frac{\pi }{2}\,[/latex]на [латекс]\,\mathrm{\theta . }[/latex]

[латекс]\ begin {массив} {l} r = 2 \ mathrm {sin} \ left (\ frac {\ pi} {2} \ right) \ hfill \\ r = 2 \ left (1 \ right) \ hfill \\ r = 2\hfill \end{массив}[/latex][/hidden-answer]

Анализ

Точка[latex]\,\left(2,\frac{\pi }{2}\right)\,[/latex] будет максимальным значением на графике. Давайте нанесем еще несколько точек, чтобы проверить график окружности. См. (Рисунок) и (Рисунок).

[латекс]\тета[/латекс] [латекс]r=2\mathrm{sin}\,\тета[/латекс] [латекс]г[/латекс]
0 [латекс]r=2\mathrm{sin}\left(0\right)=0[/latex] [латекс]0[/латекс]
[латекс]\frac{\pi }{6}[/латекс] [латекс] r = 2 \ mathrm {sin} \ left (\ frac {\ pi {6} \ right) = 1 [/latex] [латекс]1[/латекс]
[латекс]\frac{\pi }{3}[/латекс] [латекс] r = 2 \ mathrm {sin} \ left (\ frac {\ pi} {3} \ right) \ приблизительно 1,73 [/ латекс] [латекс]1,73[/латекс]
[латекс]\frac{\pi }{2}[/латекс] [латекс] r = 2 \ mathrm {sin} \ left (\ frac {\ pi} {2} \ right) = 2 [/latex] [латекс]2[/латекс]
[латекс]\frac{2\pi }{3}[/латекс] [латекс]r=2\mathrm{sin}\left(\frac{2\pi }{3}\right)\приблизительно 1,73[/latex] [латекс]1,73[/латекс]
[латекс]\frac{5\pi }{6}[/латекс] [латекс]r=2\mathrm{sin}\left(\frac{5\pi }{6}\right)=1[/latex] [латекс]1[/латекс]
[латекс]\pi[/латекс] [латекс]r=2\mathrm{sin}\left(\pi \right)=0[/latex] [латекс]0[/латекс]
Рисунок 4.

Попробуйте

Без преобразования в декартовы координаты проверьте данное уравнение на симметрию и найдите нули и максимальные значения [латекс]\,|r|:\,[/латекс][латекс]\, r=3\mathrm{cos}\,\theta .[/latex]

[reveal-answer q=”fs-id1165135309771″]Show Solution[/reveal-answer]
[hidden-answer a=”fs-id1165135309771 ″]

Тесты выявят симметрию относительно полярной оси. Нуль равен[латекс]\,\влево(0,\фракция{\pi }{2}\право),\,[/латекс], а максимальное значение равно[латекс]\,\влево(3,0\вправо). ).[/латекс]

[/hidden-answer]

Исследование кругов

Теперь мы рассмотрели уравнение окружности в полярной системе координат. В последних двух примерах одно и то же уравнение использовалось для иллюстрации свойств симметрии и демонстрации того, как находить нули, максимальные значения и точки на графике, по которым строятся графики. Однако круг — это лишь одна из многих фигур в наборе полярных кривых.

Есть пять классических полярных кривых : кардиоиды , лимаҫоны, лемнискаты, кривые розы и Спирали Архимеда . Мы кратко коснемся полярных формул для окружности, прежде чем перейти к классическим кривым и их вариациям.

Формулы для уравнения окружности

Некоторые формулы для построения графика окружности в полярных координатах имеют вид[latex]\,r=a\mathrm{cos}\,\theta \,[/latex ]and[латекс]\,r=a\mathrm{sin}\,\theta ,[/latex], где [латекс]\,a\,[/latex] — диаметр окружности или расстояние от полюса до самая дальняя точка окружности. Радиус равен [латекс]\,\фрак{|а|}{2},[/латекс] или половине диаметра. Для [латекс]\,r=a\mathrm{cos}\,\theta , [/latex] центр [латекс]\,\left(\frac{a}{2},0\right).\, [/latex]Для [латекса]\,r=a\mathrm{sin}\,\theta ,[/latex] центр [латекс]\,\left(\frac{a}{2},\frac{ \pi }{2}\right).\,[/latex](Рисунок) показывает графики этих четырех окружностей.

Рисунок 5.

Набросок графика полярного уравнения для окружности

Нарисуйте график [latex]\,r=4\mathrm{cos}\,\theta .[/latex]

[reveal- ответ q=”fs-id1165137606189″]Показать решение[/reveal-answer]
[скрытый-ответ a=”fs-id1165137606189″]

Во-первых, проверяя уравнение симметрии, мы обнаруживаем, что график симметричен относительно полярной оси. Далее находим нули и максимум[латекс]\,|r|\,[/латекс]для[латекс]\,r=4\mathrm{cos}\,\theta .\,[/латекс] Во-первых, устанавливаем [латекс]\,r=0,\,[/латекс] и найти [латекс]\,\тета[/латекс]. Таким образом, ноль находится в точке [латекс]\,\тета =\фракция{\пи} {2}±k\пи .\,[/латекс]. \text{​}\text{​}\frac{\pi }{2}\right)\,.[/latex]

Чтобы найти максимальное значение [latex]\,r,[/latex], обратите внимание, что максимальное значение функции косинуса равно 1, когда [latex]\,\theta =0±2k\pi .\,[/latex ] Подставьте [латекс]\,\тета =0\,[/латекс]в уравнение:

[латекс]\начало{массив}{с}r=4\mathrm{cos}\,\тета \\ \, \,\,\,\,r=4\mathrm{cos}\left(0\right)\\ \,\,\,\,\,\,\,r=4\left(1\right)= 4\end{array}[/latex]

Максимальное значение уравнения равно 4. Ключевой точкой для построения является [latex]\,\left(4,\,0\right).[/latex]

Поскольку [латекс]\,r=4\mathrm{cos}\,\theta \,[/латекс] симметричен относительно полярной оси, нам нужно вычислить только r -значения для [латекс]\,\тета \,[/латекс] на интервале [латекс]\,\слева[0,\,\,\пи \справа]. \,[/латекс]Точки в тогда верхний квадрант может быть отражен в нижнем квадранте. Составьте таблицу значений, аналогичную (рисунок). График показан на (рисунок).

[латекс]\тета[/латекс] 0 [латекс]\frac{\pi }{6}[/латекс] [латекс]\frac{\pi }{4}[/латекс] [латекс]\frac{\pi }{3}[/латекс] [латекс]\frac{\pi }{2}[/латекс] [латекс]\frac{2\pi }{3}[/латекс] [латекс]\frac{3\pi }{4}[/латекс] [латекс]\frac{5\pi }{6}[/латекс] [латекс]\pi[/латекс]
[латекс]r[/латекс] 4 3,46 2,83 2 0 −2 −2,83 −3,46 4
Рисунок 6.

[/hidden-answer]

Исследование кардиоид

Хотя перевод из полярных координат в декартовы координаты в некоторых случаях может показаться более простым, построение классических кривых на самом деле менее сложно в полярной системе. Следующая кривая называется кардиоидой, так как напоминает сердце. Эту форму часто включают в семейство кривых, называемых лимасонами, но здесь мы обсудим кардиоиду отдельно.

Формулы для кардиоиды

Формулы, по которым строятся графики кардиоиды, задаются как [латекс]\,r=a±b\mathrm{cos}\,\theta \,[/latex]и[латекс]\ ,r=a±b\mathrm{sin}\,\theta\,[/latex],где[латекс]\,a>0,\,\,b>0,\,[/latex]и[латекс]\ ,\frac{a}{b}=1.\,[/latex]Кардиоидный график проходит через полюс, как мы видим на (рис.).

Рисунок 7.

Как сделать

Имея полярное уравнение кардиоиды, нарисуйте ее график.

  1. Проверьте уравнение для трех типов симметрии.
  2. Найдите нули. Установить[латекс]\,r=0.[/латекс]
  3. Найдите максимальное значение уравнения по максимальному значению тригонометрического выражения.
  4. Создайте таблицу значений для [латекс]\,r\,[/латекс] и [латекс]\,\тета .[/латекс]
  5. Нанесите точки и нарисуйте график.

Набросок графика кардиоиды

Набросок графика [латекс]\,r=2+2\mathrm{cos}\,\theta .[/latex]

[reveal-answer q=»fs-id1165134061986 ″]Показать решение[/reveal-answer]
[скрытый-ответ a=”fs-id1165134061986″]

Сначала, проверяя уравнение на симметрию, мы обнаруживаем, что график этого уравнения будет симметричен относительно полярной оси. Далее находим нули и максимумы. Установив [латекс]\,r=0,\,[/латекс]имеем[латекс]\,\тета =\пи +2к\пи .\,[/латекс]Нуль уравнения расположен в [латекс] \,\left(0,\pi \right).\,[/latex]График проходит через эту точку.

Максимальное значение [латекс]\,r=2+2\mathrm{cos}\,\theta \,[/latex] имеет место, когда [латекс]\,\mathrm{cos}\,\theta \,[ /latex] является максимальным, когда[latex]\,\mathrm{cos}\,\theta =1\,[/latex]или когда[latex]\,\theta =0. \,[/latex] Подставьте [латекс]\,\тета =0\,[/латекс]в уравнение и решите для[латекс]\,р.\,[/латекс]

[латекс]\начало{массив}{л}\ begin{array}{l}\\ r=2+2\mathrm{cos}\left(0\right)\end{array}\hfill \\ r=2+2\left(1\right)=4\ hfill \end{array}[/latex]

Точка[latex]\,\left(4,0\right)\,[/latex] является максимальным значением на графике.

Мы обнаружили, что полярное уравнение симметрично относительно полярной оси, но, поскольку оно распространяется на все четыре квадранта, нам нужно отобразить значения на интервале [латекс]\,\влево[0,\,\пи \вправо ].\,[/latex]Верхняя часть графика затем отражается по полярной оси. Далее составляем таблицу значений, как на (Рисунок), а затем наносим точки и рисуем график. См. (Рисунок).

[латекс]\тета[/латекс] [латекс]0[/латекс] [латекс]\frac{\pi }{4}[/латекс] [латекс]\frac{\pi }{2}[/латекс] [латекс]\frac{2\pi }{3}[/латекс] [латекс]\pi[/латекс]
[латекс]r[/латекс] 4 3,41 2 1 0
Рисунок 8.

[/hidden-answer]

Изучение Лимасона

Слово limaçon на старофранцузском языке означает «улитка» — название, описывающее форму графика. Как упоминалось ранее, кардиоида является членом семейства лимасонов, и мы можем видеть сходство на графиках. Другие изображения в этой категории включают лимасон с одной петлей и лимасон с двумя петлями (или внутренней петлей). Лимасоны с одной петлей иногда называют лимасонами с ямочками, когда[латекс]\,1

Формулы для лимасона с одной петлей

Формулы для построения графика однопетлевого лимасона с ямочками задаются как [латекс]\,r=a±b\mathrm{cos}\,\theta \,[/latex ] и [латекс]\,r=a±b\mathrm{sin}\,\theta \,[/latex], где [латекс]\,a>0,\,b>0,\,\,\text{ и 1 (рисунок).

Рисунок 9. Лимасон с ямочками

Как сделать

Имея полярное уравнение для лимасона с одной петлей, нарисуйте график.

  1. Проверьте уравнение на симметрию. Помните, что провал теста на симметрию не означает, что форма не будет демонстрировать симметрию. Часто симметрия может проявляться при нанесении точек.
  2. Найдите нули.
  3. Найдите максимальные значения по тригонометрическому выражению.
  4. Сделать стол.
  5. Нанесите точки и нарисуйте график.

Набросок графика лимасона с одной петлей

Нарисуйте уравнение[latex]\,r=4-3\mathrm{sin}\,\theta .[/latex]

[открыть-ответ q=»fs-id1165134532781″]Показать решение[/открыть-ответ]
[скрытый-ответ a=»fs-id1165134532781″]

Во-первых, проверяя уравнение на симметрию, мы обнаруживаем, что оно не проходит все три теста на симметрию, а это означает, что граф может демонстрировать или не демонстрировать симметрию, поэтому мы не можем использовать симметрию, чтобы построить его. Однако это уравнение имеет график, который четко демонстрирует симметрию относительно линии [латекс]\,\тета =\фракция{\пи} {2},\,[/латекс], но не проходит все три теста на симметрию. Графический калькулятор сразу же проиллюстрирует отражающую способность графика.

Затем мы находим нули и максимум и наносим точки отражения для проверки симметрии. Установка [латекс]\,r=0\,[/латекс] приводит к тому, что [латекс]\,\тета \,[/латекс] не определен. Что это значит? Как [латекс]\,\тета \,[/латекс] может быть неопределенным? Угол[латекс]\,\тета\,[/латекс] не определен для любого значения [латекс]\,\mathrm{sin}\,\тета>1.\,[/латекс]Поэтому,[латекс]\ ,\theta \,[/latex] не определен, потому что не существует значения [latex]\,\theta \,[/latex], для которого [latex]\,\mathrm{sin}\,\theta >1.\ ,[/latex]Следовательно, график не проходит через полюс. Возможно, график пересекает полярную ось, но не на полюсе. Мы можем исследовать другие перехваты, вычислив [латекс]r[/латекс], когда [латекс]\,\тета =0.\,[/латекс]

[латекс]\begin{array}{l}r\left(0\right)=4-3\mathrm{sin}\left(0\right)\hfill \\ \,\,\,\,\ ,\,\,r=4-3\cdot 0=4\hfill \end{array}[/latex]

Таким образом, в точке [latex]\,\left(4,0) есть хотя бы одна точка пересечения полярной оси \right). [/latex]

Далее, поскольку максимальное значение функции синуса равно 1, когда [latex]\,\theta =\frac{\pi }{2},\,[/latex] мы подставим [latex]\,\theta =\frac{\pi }{2}\,[/latex]
в уравнение и решить для[latex]\,r.\,[/latex] Таким образом,[latex]\, г=1.[/латекс]

Составьте таблицу координат, аналогичную (Рисунок).

[латекс]\тета[/латекс] [латекс]0[/латекс] [латекс]\frac{\pi }{6}[/латекс] [латекс]\frac{\pi }{3}[/латекс] [латекс]\frac{\pi }{2}[/латекс] [латекс]\frac{2\pi }{3}[/латекс] [латекс]\frac{5\pi }{6}[/латекс] [латекс]\pi[/латекс] [латекс]\frac{7\pi }{6}[/латекс] [латекс]\frac{4\pi }{3}[/латекс] [латекс]\frac{3\pi }{2}[/латекс] [латекс]\frac{5\pi }{3}[/латекс] [латекс]\frac{11\pi }{6}[/латекс] [латекс]2\пи[/латекс]
[латекс]r[/латекс] 4 2,5 1,4 1 1,4 2,5 4 5,5 6,6 7 6,6 5,5 4

График показан на (Рисунок).

Рисунок 10.

[/hidden-answer]

Анализ

Это пример кривой, для которой создание таблицы значений имеет решающее значение для построения точного графика. Тесты на симметрию терпят неудачу; ноль не определен. Хотя может быть очевидным, что уравнение, включающее [латекс]\,\mathrm{sin}\,\theta \,[/latex], скорее всего, симметрично относительно линии [латекс]\,\theta =\frac{\pi {2},[/latex] оценка большего количества баллов помогает проверить правильность графика.

Попробуй

Нарисуйте график [latex]\,r=3-2\mathrm{cos}\,\theta .[/latex]

[reveal-answer q=»fs-id1165135159863″]Покажите решение[/reveal- ответ]
[скрытый-ответ a=”fs-id1165135159863″][/hidden-answer]

Другой тип лимасона, лимасон с внутренней петлей , назван в честь петли, образованной внутри общей формы лимасона. Он был открыт немецким художником Альбрехтом Дюрером (1471–1528), который раскрыл метод рисования лимасона внутренней петли в своей книге 1525 года Underweysung der Messing 9. 0004 . Столетие спустя отец математика Блеза Паскаля, Этьен Паскаль (1588-1651), заново открыл его.

Формулы для лимасонов внутреннего цикла

Формулы, порождающие лимасоны внутреннего цикла, задаются как [латекс]\,r=a±b\mathrm{cos}\,\theta \,[/latex]и[латекс ]\,r=a±b\mathrm{sin}\,\theta \,[/latex],где[латекс]\,a>0,\,\,b>0,\,[/latex]и[латекс ]\,\,a(Рисунок) для графиков.

Рисунок 11.

Набросок графика внутреннего цикла Limaçon

Набросок графика [latex]\,r=2+5\text{cos}\,\theta .[/latex]

[открыть-ответ q=»fs-id1165135196653″]Показать решение[/открыть-ответ]
[скрытый-ответ a=»fs-id1165135196653″]

Проверка на симметрию показывает, что график уравнения симметричен относительно полярной оси. Далее, нахождение нулей показывает, что когда [латекс]\,r=0,\,\,\тета =1,98.\,[/латекс]
Максимум[латекс]\,|r|\,[/латекс] равен найдено, когда [латекс]\,\mathrm{cos}\,\theta =1\,[/latex]или когда[латекс]\,\theta =0. \,[/latex] Таким образом, максимум находится на точка (7, 0).

Несмотря на то, что мы нашли симметрию, ноль и максимум, нанесение дополнительных точек поможет определить форму, и тогда появится закономерность.

См. (рисунок).

[латекс]\тета[/латекс] [латекс]0[/латекс] [латекс]\frac{\pi }{6}[/латекс] [латекс]\frac{\pi }{3}[/латекс] [латекс]\frac{\pi }{2}[/латекс] [латекс]\frac{2\pi }{3}[/латекс] [латекс]\frac{5\pi }{6}[/латекс] [латекс]\pi[/латекс] [латекс]\frac{7\pi }{6}[/латекс] [латекс]\frac{4\pi }{3}[/латекс] [латекс]\frac{3\pi }{2}[/латекс] [латекс]\frac{5\pi }{3}[/латекс] [латекс]\frac{11\pi }{6}[/латекс] [латекс]2\пи[/латекс]
[латекс]r[/латекс] 7 6,3 4,5 2 −0,5 −2,3 −3 −2,3 −0,5 2 4,5 6,3 7

Как и ожидалось, значения начинают повторяться после [latex]\,\theta =\pi . {-1}0=\frac{\pi }{2}\hfill & \hfill \\ \,\,\,\,\,\,\,\,\,\,\,\,\,\ ,\,u=\frac{\pi }{2}\hfill & \text{Замените}2\theta \text{ обратно на }u.\hfill \\ \,\,\,\,\,\, \,\,\,\,\,\,2\theta =\frac{\pi }{2}\hfill & \hfill \\ \,\,\,\,\,\,\,\,\, \,\,\,\,\,\,\theta =\frac{\pi }{4}\hfill & \hfill \end{массив}[/latex] 9{2}=4\влево(1\вправо)=4\\ \,\,\,\,\,\,\,\,\,\,r=±\sqrt{4}\,=2\конец {array}[/latex]

У нас есть максимум в (2, 0). Поскольку этот график симметричен относительно полюса, линии [латекс]\,\тета =\фракция{\пи} {2},[/латекс] и полярной оси, нам нужно нанести только точки в первом квадранте. .

Составьте таблицу, аналогичную (рисунок).

[латекс]\тета[/латекс] 0 [латекс]\frac{\pi }{6}[/латекс] [латекс]\frac{\pi }{4}[/латекс] [латекс]\frac{\pi }{3}[/латекс] [латекс]\frac{\pi }{2}[/латекс]
[латекс]r[/латекс] 2 [латекс]\sqrt{2}[/латекс] 0 [латекс]\sqrt{2}[/латекс] 0

Нанесите точки на график, как показано на (Рисунок).

Рисунок 14. Лемниската

[/hidden-answer]

Анализ

Выполнение замены, такой как [латекс]\,u=2\тета \,[/латекс] является обычной практикой в ​​математике, потому что это может сделать расчеты проще. Однако мы не должны забывать заменить термин замены исходным термином в конце, а затем найти неизвестное.

Некоторые точки на этом графике могут не отображаться при использовании функции Trace на графическом калькуляторе TI-84, а таблица калькулятора может отображать ошибку для этих же точек [latex]\,r.\,[/latex ]Это потому, что для этих значений [латекс]\,\тета .\,[/латекс] нет реальных квадратных корней. Другими словами, соответствующие r -значения [латекс]\,\sqrt{4\ mathrm{cos}\left(2\theta \right)}\,[/latex]
являются комплексными числами, потому что под радикалом стоит отрицательное число.

Исследование кривых розы

Следующий тип полярного уравнения дает форму лепестка, называемую кривой розы. Хотя графики выглядят сложными, простое полярное уравнение создает закономерность.

Кривые розы

Формулы, которые создают график кривой розы, задаются как [латекс]\,r=a\mathrm{cos}\,n\theta \,[/latex] и [латекс]\,r =a\mathrm{sin}\,n\theta \,[/latex],где[latex]\,a\ne 0.\,[/latex]Если[latex]\,n\,[/latex]четно , кривая имеет[латекс]\,2n\,[/латекс]лепестки. Если [латекс]\,n\,[/латекс]нечетный, кривая имеет[латекс]\,n\,[/латекс]лепестки. См. (Рисунок).

Рисунок 15.

Набросок графика кривой розы (

n Четный)

Нарисуйте график [латекс]\,r=2\mathrm{cos}\,4\theta .[/latex]

[reveal-answer q=»6

″]Показать решение[/reveal-answer]
[hidden-answer a=»6
″]

Проверяя симметрию, мы снова обнаруживаем, что проверки симметрии не дают полной картины. График симметричен не только относительно полярной оси, но и относительно линии [латекс]\,\тета =\фракция{\пи} {2}\,[/латекс] и полюса. 9{-1}0=u\\ \,\,\,\,\,\,\,\,\,\,\,\,\,\,\,\,\,u=\frac{\pi {2}\\ \,\,\,\,\,\,\,\,\,\,\,\,\,\,4\theta =\frac{\pi }{2}\\ \ ,\,\,\,\,\,\,\,\,\,\,\,\,\,\,\,\,\,\theta =\frac{\pi }{8}\end{ array}[/latex]

Нуль равен [латекс]\,\theta =\frac{\pi }{8}. \,[/latex]Точка[латекс]\,\left(0,\frac{ \pi }{8}\right)\,[/latex] находится на кривой.

Далее находим максимальное[латекс]\,|r|.\,[/латекс]Мы знаем, что максимальное значение[латекс]\,\mathrm{cos}\,u=1\,[/латекс ]когда[латекс]\,\тета =0.\,[/латекс] Таким образом,

[латекс]\begin{array}{l}\\ \begin{array}{l}r=2\mathrm{cos}\left(4\cdot 0\right)\hfill \\ r=2\mathrm {cos}\left(0\right)\hfill \\ r=2\left(1\right)=2\hfill \end{массив}\end{массив}[/latex]

Точка[латекс]\ ,\left(2,0\right)\,[/latex] находится на кривой.

График кривой розы обладает уникальными свойствами, которые раскрываются на (Рисунок).

[латекс]\тета[/латекс] 0 [латекс]\frac{\pi }{8}[/латекс] [латекс]\frac{\pi }{4}[/латекс] [латекс]\frac{3\pi }{8}[/латекс] [латекс]\frac{\pi }{2}[/латекс] [латекс]\frac{5\pi }{8}[/латекс] [латекс]\frac{3\pi }{4}[/латекс]
[латекс]r[/латекс] 2 0 −2 0 2 0 −2

Так как [латекс]\,r=0\,[/латекс]когда [латекс]\,\тета =\frac{\pi }{8},\,[/латекс]имеет смысл делить значения в таблице на [латекс]\,\frac{\pi }{8}\,[/латекс]единицы. Возникает определенная закономерность. Посмотрите на диапазон r -значения: 2, 0, −2, 0, 2, 0, −2 и т. д. Это представляет развитие кривой по одному лепестку за раз. Начиная с [латекс]\,r=0,\,[/латекс]каждый лепесток простирается на расстояние [латекс]\,r=2,\,[/латекс] и затем возвращается к нулю[латекс]\, 2n\,[/latex]раз, всего восемь лепестков. См. график на (рис.).

Рисунок 16. Кривая Розы,[латекс]\,n\,[/латекс]эвен

[/hidden-answer]

Анализ

как на (рис.). Это позволяет нам увидеть, как график достигает максимума (кончик лепестка), зацикливается на пересечении полюса, достигает противоположного максимума и возвращается к полюсу. Действие продолжается до тех пор, пока не будут нарисованы все лепестки.

Попробуйте

Нарисуйте график [латекс]\,r=4\mathrm{sin}\left(2\theta\right).[/latex]

[reveal-answer q=»fs-id1165137638079″ ]Показать решение[/reveal-answer]
[скрытый-ответ a=”fs-id1165137638079″]

График представляет собой кривую розы,[латекс]\,n\,[/латекс]четный

[/hidden-answer]

Набросок графика кривой розы (

n Нечетный)

Набросок графика of[latex]\,r=2\mathrm{sin}\left(5\theta\right). {- 1}0=0\\ \,\,\,\,\,\ ,\,\,\,\,\,\,\,\,u=0\\ \,\,\,\,\,\,\,\,\,\,\,5\тета =0\ \ \,\,\,\,\,\,\,\,\,\,\,\,\,\,\theta =0\end{массив}[/латекс]

Максимальное значение вычисляется для угла, где [латекс]\,\mathrm{sin}\,\theta \,[/латекс] является максимальным. Следовательно,

[латекс]\begin{array}{l}\begin{array}{l}\\ r=2\mathrm{sin}\left(5\cdot \frac{\pi }{2}\right )\end{array}\hfill \\ r=2\left(1\right)=2\hfill \end{array}[/latex]

Таким образом, максимальное значение полярного уравнения равно 2. Это длина каждого лепестка. Поскольку кривая для [латекс]\,n\,[/латекс]нечет дает то же количество лепестков, что и [латекс]\,n,\,[/латекс], на графике будет пять лепестков. См. (Рисунок).

Создайте таблицу значений, аналогичную (рисунок).

«>
[латекс]\тета[/латекс] 0 [латекс]\frac{\pi }{6}[/латекс] [латекс]\frac{\pi }{3}[/латекс] [латекс]\frac{\pi }{2}[/латекс] [латекс]\frac{2\pi }{3}[/латекс] [латекс]\frac{5\pi }{6}[/латекс] [латекс]\pi[/латекс]
[латекс]r[/латекс] 0 1 −1,73 2 −1,73 1 0
Рисунок 17. Кривая розы,[latex]\,n\,[/latex]нечетный

[/hidden-answer]

Попробуйте

Нарисуйте график [latex]r=3\mathrm{ cos}\left(3\theta \right).[/latex]

[reveal-answer q=”fs-id1165135245746″]Показать решение[/reveal-answer]
[скрытый-ответ a=”fs-id1165135245746″ ]

Кривая Розы,[латекс]\,n\,[/латекс]нечетный

[/hidden-answer]

Исследование спирали Архимеда

Последнее полярное уравнение, которое мы обсудим, — это спираль Архимеда, названная в честь ее первооткрывателя, греческого математика Архимеда (ок. 287 г. до н. э. — ок. 212 г. до н. э.), которому приписывают многочисленные открытия в области геометрии и механики.

Спираль Архимеда

Формула, по которой построен график спирали Архимеда, имеет вид [латекс]\,r=\theta \,[/latex]
for[latex]\,\theta \ge 0.\ ,[/latex]По мере того, как[латекс]\,\тета\,[/латекс]увеличивается,[латекс]\,r\,[/латекс]
увеличивается с постоянной скоростью в постоянно расширяющейся, бесконечной, спиральной дорожка. См. (Рисунок).

Рисунок 18.

Как сделать

Дана спираль Архимеда над [латексом]\,\слева[0,2\пи \справа],[/латекс]нарисуйте график.

  1. Составьте таблицу значений для [латекс]\,r\,[/латекс] и [латекс]\,\тета \,[/латекс] для заданного домена.
  2. Нанесите точки и нарисуйте график.

Набросок графика спирали Архимеда

Нарисуйте график [латекс]\,r=\theta \,[/latex]над [латексом]\,\left[0,2\pi \right].[ /латекс]

[reveal-answer q=»fs-id1165137755308″]Показать решение[/reveal-answer]
[скрытый-ответ a=»fs-id1165137755308″]

Так как [латекс]\,r\,[/латекс] равно [латекс]\,\тета ,\,[/латекс]участок спирали Архимеда начинается на полюсе в точке (0, 0 ). Хотя график намекает на симметрию, формальной симметрии в отношении прохождения тестов на симметрию нет. Кроме того, максимальное значение отсутствует, если домен не ограничен.

Создайте таблицу, например (рисунок).

[латекс]\тета[/латекс] [латекс]\frac{\pi }{4}[/латекс] [латекс]\frac{\pi }{2}[/латекс] [латекс]\pi[/латекс] [латекс]\frac{3\pi }{2}[/латекс] [латекс]\frac{7\pi }{4}[/латекс] [латекс]2\пи[/латекс]
[латекс]r[/латекс] 0,785 1,57 3,14 4,71 5,50 6,28

Обратите внимание, что значения r — это просто десятичная форма угла, измеренного в радианах. Мы можем видеть их на графике в (Рисунок).

Рисунок 19. Спираль Архимеда

[/hidden-answer]

Анализ

Область определения этой полярной кривой: [латекс]\,\влево[0,2\пи \вправо].\,[/латекс ]В общем случае, однако, область определения этой функции равна[latex]\,\left(-\infty ,\infty \right).\,[/latex]Графическое уравнение спирали Архимеда довольно просто, хотя изображение создает впечатление, что оно сложное.

Попробуйте

Нарисуйте график [латекс]\,r=-\тета \,[/латекс] на интервале [латекс]\,\слева[0,4\пи \справа].[/латекс]

[reveal-answer q=”fs-id1165134338802″]Show Solution[/reveal-answer]
[hidden-answer a=”fs-id1165134338802″][/hidden-answer]

Сводка кривых

0, b >0, а/б=1. (C) — лимасоны с одной петлей. r= a + или – bcos(тета), или r= a + или – bsin(тета). a>0, b>0, 10, b>0, a Рис. 20.

В этом разделе мы исследовали ряд кажущихся сложными полярных кривых. (Рисунок) и (Рисунок) суммируют графики и уравнения для каждой из этих кривых.

Рисунок 21.

Доступ к этим онлайн-ресурсам для получения дополнительных инструкций и практики с графиками полярных координат.

  • Графики полярных уравнений, часть 1
  • Графики полярных уравнений, часть 2
  • Анимация: графики полярных уравнений
  • Графики полярных уравнений на TI-84

Ключевые понятия

  • Начертить полярные уравнения проще, если мы можем проверить уравнения на симметрию относительно линии[латекс]\,\тета =\фракция{\пи} {2},\,[/латекс ] полярная ось или полюс.
  • Существует три теста на симметрию, которые показывают, будет ли график полярного уравнения демонстрировать симметрию. Если уравнение не проходит тест на симметрию, график может демонстрировать или не демонстрировать симметрию. См. (Рисунок).
  • Полярные уравнения можно представить в виде графика, составив таблицу значений для [латекс]\,\тета\,[/латекс] и [латекс]\,r. [/латекс]
  • Максимальное значение полярного уравнения находится путем подстановки значения [латекс]\,\тета \,[/латекс], которое приводит к максимальному значению тригонометрического выражения.
  • Нули полярного уравнения можно найти, установив [латекс]\,r=0\,[/латекс] и найдя [латекс]\,\тета .\,[/латекс] См. (рисунок).
  • Некоторые формулы, которые создают график окружности в полярных координатах, задаются как [латекс]\,r=a\mathrm{cos}\,\theta \,[/latex]и [латекс]\,r=a\mathrm {sin}\,\theta .\,[/latex] См. (рисунок).
  • Формулы, по которым строятся графики кардиоиды, задаются как [латекс]\,r=a±b\mathrm{cos}\,\theta \,[/latex]и[латекс]\,r=a±b\ mathrm{sin}\,\theta ,\,[/latex]для[латекса]\,a>0,\,\,b>0,\,[/latex]и[латекс]\,\frac{a} {b}=1.\,[/latex] См. (Рисунок).
  • Формулы, по которым строятся графики лимасона с одной петлей, задаются как [латекс]\,r=a±b\mathrm{cos}\,\theta \,[/latex]и[латекс]\,r=a ±b\mathrm{sin}\,\theta \,[/latex]for[latex]\,1 (рисунок). {2}\mathrm{sin}\,2\theta ,\,[/latex],где[латекс]\,a\ne 0.[/ латекс] См. (Рисунок).
  • Формулы, по которым строятся графики розовых кривых, задаются как [латекс]\,r=a\mathrm{cos}\,n\theta \,[/latex]и[латекс]\,r=a\mathrm{sin }\,n\theta ,\,[/latex]где[латекс]\,a\ne 0;\,[/latex]если [латекс]\,n\,[/latex]четно, существуют[латекс ]\,2n\,[/latex]лепестков, а если [латекс]\,n\,[/латекс]нечётно, то [латекс]\,n\,[/латекс]лепестков. См. (Рисунок) и (Рисунок).
  • Формула, по которой строится график спирали Архимеда, имеет вид[latex]\,r=\theta ,\,\,\theta \ge 0.\,[/latex]См. (рисунок).

Упражнения по разделам

Вербальные

Опишите три типа симметрии в полярных графах и сравните их с симметрией декартовой плоскости.

[reveal-answer q=»fs-id1165137806288″]Показать решение[/reveal-answer]
[скрытый-ответ a=»fs-id1165137806288″]

Симметрия относительно полярной оси аналогична симметрии относительно оси [латекс]\,x[/латекс], симметрия относительно полюса аналогична симметрии относительно начала координат и симметрична относительно линии[ латекс]\,\theta =\frac{\pi }{2}\,[/latex] аналогичен симметрии относительно оси [latex]\,y[/latex].

[/hidden-answer]

Какие из трех типов симметрий для полярных графов соответствуют симметриям относительно оси x , оси y и начала координат?

Какие шаги необходимо выполнить при построении графика полярных уравнений?

[reveal-answer q=»fs-id1165135195812″]Показать решение[/reveal-answer]
[скрытый-ответ a=»fs-id1165135195812″]

Тест на симметрию; найти нули, точки пересечения и максимумы; составить таблицу значений. Определите общий тип графика, кардиоида, лимасон, лемнискат и т. д., затем нанесите точки на [латекс]\,\тета = 0,\,\frac{\pi }{2},\,\,\pi \, \,\text{и }\frac{3\pi }{2},\,[/latex] и нарисуйте график.

[/hidden-answer]

Опишите формы графиков кардиоид, лимасонов и лемнискат.

Какая часть уравнения определяет форму графика полярного уравнения?

[reveal-answer q=»fs-id1165137565862″]Показать решение[/reveal-answer]
[hidden-answer a=»fs-id1165137565862″]

Форма полярной диаграммы определяется тем, включает ли она синус, косинус и константы в уравнение.

[/скрытый ответ]

Графический

В следующих упражнениях проверьте уравнение симметрии.

[латекс]r=5\mathrm{cos}\,3\theta[/latex]

[латекс]r=3-3\mathrm{cos}\,\theta[/latex]

[reveal- ответ q=”fs-id1165132957162″]Показать решение[/reveal-answer]
[скрытый-ответ a=”fs-id1165132957162″]симметричный относительно полярной оси[/hidden-answer]

[латекс]r= 3+2\mathrm{sin}\,\theta[/latex]

[latex]r=3\mathrm{sin}\,2\theta[/latex]

[reveal-answer q=»fs-id1165131863074 ″]Показать решение[/reveal-answer]
[скрытый ответ a=”fs-id1165131863074″]

симметричный относительно полярной оси, симметричный относительно линии [латекс]\theta =\frac{\pi }{2},[/латекс] симметричный относительно полюса

[/hidden-answer]

[latex]r=4[/latex]

[latex]r=2\theta[/latex]

[reveal-answer q=»fs-id1165135185163″]Показать решение[/reveal-answer]
[ скрытый ответ a = «fs-id1165135185163»]

нет симметрии

[/hidden-answer]

[латекс]r=4\mathrm{cos}\,\frac{\theta}{2}[/latex] 9{2}\theta }[/latex]

[latex]r=\sqrt{5\mathrm{sin}\,2\theta }[/latex]

[reveal-answer q=”fs-id1165135370170″] Показать решение[/reveal-answer]
[скрытый-ответ a=”fs-id1165135370170″]

симметрично относительно полюса

[/hidden-answer]

Для следующих упражнений нарисуйте полярное уравнение. Определите название фигуры.

[латекс]r=3\mathrm{cos}\,\theta[/latex]

[латекс]r=4\mathrm{sin}\,\theta[/latex]

[reveal-answer q= ”fs-id1165137

  • 4″]Показать решение[/reveal-answer]
    [скрытый-ответ a=”fs-id1165137
  • 4″]

    круг[/скрытый-ответ]

    [латекс]r=2+2\mathrm{cos}\,\theta[/latex]

    [латекс]r=2-2\mathrm{cos}\,\ theta[/latex]

    [reveal-answer q=”508959″]Показать решение[/reveal-answer]
    [hidden-answer a=”508959″]

    кардиоида

    [/hidden-answer]

    [латекс]r=5-5\mathrm{sin}\,\theta[/latex]

    [латекс]r=3+3\mathrm{sin}\,\theta[/latex]

    [открыть-ответ q=»fs-id1165135697823″]Показать решение[/открыть-ответ]
    [скрытый-ответ a=»fs-id1165135697823″]

    кардиоида[/hidden-answer]

    [латекс]r=3+2\mathrm{sin}\,\theta[/latex]

    [латекс]r=7+4\mathrm{sin}\,\ theta[/latex]

    [reveal-answer q=»952295″]Показать решение[/reveal-answer]
    [hidden-answer a=»952295″]лимасон с одной петлей/ямочками[/hidden-answer]

    [латекс]r=4+3\mathrm{cos}\,\theta[/latex]

    [латекс]r=5+4\mathrm{cos}\,\theta[/latex]

    [reveal-answer q=»fs-id1165134272124″]Показать решение[/reveal-answer]
    [скрытый-ответ a=»fs-id1165134272124″]

    лимасон с одной петлей/ямочками

    [/hidden-answer]

    [латекс]r=10+9\mathrm{cos}\,\theta[/латекс]

    [латекс]r=1+3\ mathrm{sin}\,\theta[/latex]

    [reveal-answer q=»fs-id1165134196056″]Show Solution[/reveal-answer]
    [hidden-answer a=»fs-id1165134196056″]

    внутренняя петля/двухпетлевая перемычка

    [/hidden-answer]

    [латекс]r=2+5\mathrm{sin}\,\theta[/latex]

    [латекс]r=5+7\mathrm{sin}\,\theta[/latex]

    [показать -answer q=”fs-id1165137663724″]Показать решение[/reveal-answer]
    [скрытый-ответ a=”fs-id1165137663724″]

    внутренняя петля/двухпетлевая лимасон

    [/hidden-answer]

    [латекс]r=2+4\mathrm{cos}\,\theta[/латекс]

    [латекс]r=5+6 \mathrm{cos}\,\theta[/latex]

    [reveal-answer q=”fs-id1165137855231″]Показать решение[/reveal-answer]
    [скрытый-ответ a=”fs-id1165137855231″] 9{2}=10\mathrm{sin}\left(2\theta \right)[/latex]

    [reveal-answer q=”fs-id1165137514792″]Показать решение[/reveal-answer]
    [скрытый-ответ a=”fs-id1165137514792″]

    лемниската

    [/hidden-answer]

    [латекс]r=3\текст{sin}\left(2\theta\right)[/latex]

    [латекс]r=3\text{cos} \left(2\theta \right)[/latex]

    [reveal-answer q=»fs-id1165135628620″]Show Solution[/reveal-answer]
    [hidden-answer a=»fs-id1165135628620″]

    кривая розы

    [/hidden-answer]

    [латекс]r=5\текст{sin}\left(3\theta\right)[/latex]

    [латекс]r=4\text{sin}\left( 4\theta \right)[/latex]

    [reveal-answer q=”fs-id1165135639747″]Show Solution[/reveal-answer]
    [hidden-answer a=”fs-id1165135639747″]

    кривая розы

    [/hidden-answer]

    [латекс]r=4\текст{sin}\left(5\theta \right)[/latex]

    [латекс]r=-\theta[/ латекс]

    [reveal-answer q=»fs-id1165135436311″]Показать решение[/reveal-answer]
    [скрытый ответ a=”fs-id1165135436311″]

    Спираль Архимеда

    [/hidden-answer]

    [latex]r=2\theta[/latex]

    [latex]r=-3\theta[/latex]

    [reveal-answer q= ”fs-id1165135664929″]Показать решение[/reveal-answer]
    [скрытый-ответ a=”fs-id1165135664929″]

    Спираль Архимеда

    [/hidden-answer]

    Технология

    В следующих упражнениях используйте графический калькулятор, чтобы нарисовать график полярного уравнения. 9{2}[/latex]

    [latex]r=\theta +1[/latex]

    [reveal-answer q=”fs-id1165137584082″]Показать решение[/reveal-answer]
    [скрытый-ответ a =”fs-id1165137584082″][/hidden-answer]

    [латекс]r=\тета \mathrm{sin}\,\тета[/латекс]

    [латекс]r=\тета \mathrm{cos}\ ,\theta[/latex]

    [reveal-answer q=”fs-id1165137583981″]Показать решение[/reveal-answer]
    [hidden-answer a=”fs-id1165137583981″][/hidden-answer]

    В следующих упражнениях используйте графическую утилиту для построения графика каждой пары полярных уравнений в области [латекс]\,\слева[0,4\пи \справа]\,[/латекс], а затем объясните различия, показанные на рисунке. графики.

    [латекс]r=\theta ,r=-\theta[/latex]

    [латекс]r=\theta ,r=\theta +\mathrm{sin}\,\theta[/latex]

    [ раскрыть-ответ q=”fs-id1165137547601″]Показать решение[/открыть-ответ]
    [скрытый-ответ a=”fs-id1165137547601″]

    Они оба спирали, но не совсем одинаковые.

    [/hidden-answer]

    [латекс]r=\mathrm{sin}\,\theta +\theta ,r=\mathrm{sin}\,\theta -\theta[/latex]

    [латекс ]r=2\mathrm{sin}\left(\frac{\theta}{2}\right),r=\theta\mathrm{sin}\left(\frac{\theta}{2}\right)[ /латекс]

    [открыть-ответ q=”fs-id1165137476109″]Показать решение[/открыть-ответ]
    [скрытый-ответ a=”fs-id1165137476109″]

    Оба графика представляют собой кривые с двумя петлями. Уравнение с коэффициентом [латекс]\,\тета \,[/латекс] имеет две петли слева, уравнение с коэффициентом 2 имеет две петли рядом. Нарисуйте их в виде графика от 0 до [латекс]\,4\пи \,[/латекс], чтобы получить лучшую картину.

    [/hidden-answer]

    [латекс]r=\mathrm{sin}\left(\mathrm{cos}\left(3\theta\right)\right)\,\,r=\mathrm{sin }\влево(3\тета\вправо)[/латекс]

    В графической утилите graph[latex]\,r=\mathrm{sin}\left(\frac{16}{5}\theta \right)\,[/latex]on[latex]\,\left [0,4\pi \right],\left[0,8\pi \right],\left[0,12\pi \right],\,[/latex]и[латекс]\,\left[0 ,16\pi \right]. \,[/latex]Опишите эффект увеличения ширины домена.

    [reveal-answer q=»fs-id1165137662107″]Показать решение[/reveal-answer]
    [скрытый-ответ a=»fs-id1165137662107″]

    При увеличении ширины домена видно больше лепестков цветка. 9{3}\,[/latex]on[latex]\,\left[0,4\pi \right].[/latex]

    С помощью графической утилиты нарисуйте каждое полярное уравнение. Объясните сходства и различия, которые вы видите на графиках.

    [латекс]\begin{array}{l}\begin{array}{l}\\ {r}_{1}=3\mathrm{sin}\left(3\theta\right)\end{array }\hfill \\ {r}_{2}=2\mathrm{sin}\left(3\theta\right)\hfill \\ {r}_{3}=\mathrm{sin}\left(3\ theta \right)\hfill \end{array}[/latex]

    [reveal-answer q=”fs-id1165133004400″]Show Solution[/reveal-answer]
    [скрытый ответ a = «fs-id1165133004400»]

    Графики трехлепестковые, кривые розы. Чем больше коэффициент, тем больше расстояние кривой от полюса.

    [/hidden-answer]

    В графической утилите нарисуйте график каждого полярного уравнения. Объясните сходства и различия, которые вы видите на графиках.

    [латекс]\begin{array}{l}\begin{array}{l}\\ {r}_{1}=3+3\mathrm{cos}\,\theta \end{array}\hfill \\ {r}_{2}=2+2\mathrm{cos}\,\theta \hfill \\ {r}_{3}=1+\mathrm{cos}\,\theta \hfill \end{ массив}[/латекс]

    В графической утилите нарисуйте график каждого полярного уравнения. Объясните сходства и различия, которые вы видите на графиках.

    [латекс]\begin{array}{l}\begin{array}{l}\\ {r}_{1}=3\theta \end{array}\hfill \\ {r}_{2} =2\theta \hfill \\ {r}_{3}=\theta \hfill \end{array}[/latex]

    [reveal-answer q=»fs-id1165134339992″]Показать решение[/reveal-answer ]
    [скрытый ответ a=”fs-id1165134339992″]

    Графики представляют собой спирали. Чем меньше коэффициент, тем туже спираль.

    [/hidden-answer]

    Расширения

    В следующих упражнениях нарисуйте каждое полярное уравнение на одном и том же наборе полярных осей и найдите точки пересечения.

    [латекс]{r}_{1}=3+2\mathrm{sin}\,\theta,\,{r}_{2}=2[/латекс]

    [латекс]{r}_ {1}=6-4\mathrm{cos}\,\theta ,\,{r}_{2}=4[/latex]

    [reveal-answer q=»fs-id1165133277564″]Показать решение[/ раскрыть-ответ]
    [скрытый-ответ a=”fs-id1165133277564″]

    [латекс]\влево(4,\frac{\pi }{3}\вправо),\влево(4,\frac{5\pi} }{3}\вправо)[/latex]

    [/hidden-answer]

    [латекс]{r}_{1}=1+\mathrm{sin}\,\theta ,\,{r}_{2}=3\mathrm{sin}\ ,\theta[/latex]

    [латекс]{r}_{1}=1+\mathrm{cos}\,\theta ,\,{r}_{2}=3\mathrm{cos}\, \theta[/latex]

    [reveal-answer q=”fs-id1165137766884″]Показать решение[/reveal-answer]
    [скрытый-ответ a=”fs-id1165137766884″]

    [латекс] \ влево (\ гидроразрыва {3} {2}, \ гидроразрыва {\ пи {3} \ вправо), \ влево (\ гидроразрыва {3} {2}, \ гидроразрыва {5 \ пи } { 3}\right)[/latex]

    [/hidden-answer]

    [латекс]{r}_{1}=\mathrm{cos}\left(2\theta\right),\,{r} _{2}=\mathrm{sin}\left(2\theta\right)[/latex] 9{2}=\mathrm{cos}\,\theta[/latex]

    [reveal-answer q=”fs-id1165137725132″]Показать решение[/reveal-answer]
    [hidden-answer a=”fs-id1165137725132 ″]

    [латекс]\left(\frac{\sqrt[4]{8}}{2},\frac{\pi }{4}\right),\,\left(\frac{\sqrt[4] {8}}{2},\frac{5\pi }{4}\right)\,[/latex] и at[latex]\,\theta =\frac{3\pi }{4},\, \,\frac{7\pi }{4}\,\,[/latex], так как [латекс]\,r\,[/latex] возводится в квадрат

    [/hidden-answer]

    [латекс]{r }_{1}=1+\mathrm{cos}\,\theta ,\,{r}_{2}=1-\mathrm{sin}\,\theta[/latex]

    Глоссарий

    Спираль Архимеда
    полярная кривая, заданная как [латекс]\,r=\theta . \,[/latex]При умножении на константу уравнение выглядит как[латекс]\,r=a\тета .\,[/латекс]Как [latex]\,r=\theta ,\,[/latex]кривая продолжает расширяться по спирали над доменом.
    кардиоидная
    член семейства лимасонских кривых, названный в честь сходства с сердцем; его уравнение задается как [латекс]\,r=a±b\mathrm{cos}\,\theta \,[/latex] and[latex]\,r=a±b\mathrm{sin}\,\theta ,\,[/латекс],где[латекс]\,\фракция{а}{b}=1[/латекс]
    г.
    выпуклый изгиб
    тип лимасона с одной петлей, представленный [латекс]\,r=a±b\mathrm{cos}\,\theta\,[/latex]и[латекс]\,r=a±b\mathrm{sin }\,\theta \,[/latex]такие, что[latex]\,\frac{a}{b}\ge 2[/latex]
    лиман с ямочками
    тип лимасона с одной петлей, представленный [латекс]\,r=a±b\mathrm{cos}\,\theta\,[/latex] и [латекс]\,r=a±b\mathrm{sin }\,\theta \,[/latex] такие, что[latex]\,1
    лимасон с внутренней петлей
    9{2}\mathrm{sin}\,2\theta,\,\,a\ne 0[/latex]
    одноконтурный лиман
    полярная кривая, представленная [латекс]\,r=a±b\mathrm{cos}\,\theta\,[/latex] и [латекс]\,r=a±b\mathrm{sin}\,\ theta \,[/latex] такой, что [latex]a>0,b>0,[/latex]и[latex]\,\frac{a}{b}>1;[/latex] может быть ямчатым или выпуклым ; не проходит через столб
    полярное уравнение
    уравнение, описывающее кривую на полярной сетке.
    кривая розы
    г.
    полярное уравнение, напоминающее цветок, заданное уравнениями [латекс]\,r=a\mathrm{cos}\,n\theta \,[/latex] и [латекс]\,r=a\mathrm{sin} \,n\theta ;\,[/latex]когда [латекс]\,n\,[/латекс] даже есть [латекс]\,2n\,[/латекс] лепестки, и кривая очень симметрична; когда [латекс]\,n\,[/латекс] нечетно, есть [латекс]n[/латекс] лепестков.

    8.4 Полярные координаты: графики — предварительное вычисление 2e

    Цели обучения

    В этом разделе вы:

    • Проверка полярных уравнений на симметрию.
    • Графики полярных уравнений по точкам.

    Планеты движутся в космосе по эллиптическим периодическим орбитам вокруг Солнца, как показано на рис. 1. Они находятся в постоянном движении, поэтому определение точного положения любой планеты допустимо только на мгновение. Другими словами, мы можем зафиксировать только мгновенное положение планеты. Это одно из применений полярных координат, представленных как (r, θ). (r, θ). Мы интерпретируем rr как расстояние от центра Солнца, а θθ — как угловой азимут планеты или ее направление от центра Солнца. В этом разделе мы сосредоточимся на полярной системе и графиках, которые создаются непосредственно из полярных координат.

    Рисунок 1 Планеты движутся по эллиптическим траекториям, вращаясь вокруг Солнца. (кредит: модификация работы NASA/JPL-Caltech)

    Проверка полярных уравнений на симметрию

    Точно так же, как прямоугольное уравнение, такое как y=x2y=x2, описывает отношение между xx и yy на декартовой сетке, полярное уравнение описывает отношение между rr и θθ на полярной сетке. Напомним, что пара координат (r, θ)(r, θ) указывает, что мы движемся против часовой стрелки от полярной оси (положительные x -ось) на угол θ, θ, и провести луч от полюса (начала координат) на rr единиц в направлении θ.θ. Все точки, удовлетворяющие полярному уравнению, находятся на графике.

    Симметрия — это свойство, которое помогает нам распознавать и строить график любого уравнения. Если уравнение имеет график, симметричный относительно оси, это означает, что если мы сложим график пополам по этой оси, часть графика с одной стороны совпадет с частью с другой стороны. Выполнив три теста, мы увидим, как применять свойства симметрии к полярным уравнениям. Далее будем использовать симметрию (помимо нанесения ключевых точек, нулей и максимумов r)r) построить график полярного уравнения.

    В первом тесте мы рассматриваем симметрию относительно линии θ=π2θ=π2 (ось y ). Мы заменяем (r,θ)(r,θ) на (−r,−θ)(−r,−θ), чтобы определить, эквивалентно ли новое уравнение исходному уравнению. Например, предположим, что нам дано уравнение r=2sinθ;r=2sinθ;

    r=2sinθ−r=2sin(−θ)Заменить(r,θ)на(−r,−θ).−r=−2sinθИдентичность: sin(−θ)=−sinθ.r=2sinθУмножить обе части на− 1.r=2sinθ−r=2sin(−θ)Заменить(r,θ)на(−r,−θ).−r=−2sinθИдентичность: sin(−θ)=−sinθ. r=2sinθУмножить обе части на− 1.

    Это уравнение обладает симметрией относительно прямой θ=π2.θ=π2.

    Во втором тесте мы рассматриваем симметрию относительно полярной оси (оси xx). Мы заменяем (r,θ)(r,θ) на (r,−θ)(r,−θ) или (−r,π−θ)(−r,π−θ), чтобы определить эквивалентность между тестируемым уравнением и оригинал. Например, предположим, что нам дано уравнение r=1−2cosθ.r=1−2cosθ.

    r=1−2cosθr=1−2cos(−θ)Заменить (r,θ) на (r,−θ).r=1−2cosθЧетное/нечетное тождествоr=1−2cosθr=1−2cos(−θ)Заменить (r,θ)with(r,−θ).r=1−2cosθЧетное/нечетное тождество

    График этого уравнения демонстрирует симметрию относительно полярной оси.

    В третьем тесте мы рассматриваем симметрию относительно полюса (начала). Мы заменяем (r, θ) (r, θ) на (−r, θ) (−r, θ), чтобы определить, эквивалентно ли тестируемое уравнение исходному уравнению. Например, предположим, что нам дано уравнение r=2sin(3θ).r=2sin(3θ).

    r=2sin(3θ)−r=2sin(3θ)r=2sin(3θ)−r=2sin(3θ)

    Уравнение не прошло тест на симметрию, но это не означает, что оно не симметрично относительно к полюсу. Прохождение одного или нескольких тестов на симметрию подтверждает, что симметрия будет отображаться на графике. Однако непрохождение тестов на симметрию не обязательно означает, что график не будет симметричным относительно линии θ=π2, θ=π2, полярной оси или полюса. В этих случаях мы можем подтвердить существование симметрии, нанеся отражающие точки на кажущуюся ось симметрии или полюс. Проверка на симметрию — это метод, который упрощает построение графиков полярных уравнений, но его применение несовершенно.

    Тесты симметрии

    Полярное уравнение описывает кривую на полярной сетке. График полярного уравнения можно оценить для трех типов симметрии, как показано на рисунке 2.

    Рисунок 2 (a) График симметричен относительно прямой θ=π2θ=π2 ( y -ось), если заменить (r,θ)(r,θ) на (−r,−θ)(−r,−θ ) дает эквивалентное уравнение. (b) График симметричен относительно полярной оси (ось x ), если заменить (r,θ)(r,θ) на (r,−θ)(r,−θ) или (−r, π−θ)(−r,π−θ) дает эквивалентное уравнение. (c) Граф симметричен относительно полюса (начала координат), если замена (r, θ)(r, θ) на (−r, θ)(−r, θ) дает эквивалентное уравнение.

    Как

    Имея полярное уравнение, проверьте его симметрию.

    1. Подставьте соответствующую комбинацию компонентов вместо (r,θ):(r,θ):(−r,−θ)(−r,−θ) для симметрии θ=π2θ=π2; (r,−θ)(r,−θ) для полярно-осевой симметрии; и (−r,θ)(−r,θ) для симметрии относительно полюса.
    2. Если результирующие уравнения эквивалентны в одном или нескольких тестах, график обеспечивает ожидаемую симметрию.

    Пример 1

    Проверка полярного уравнения на симметрию

    Проверка уравнения r=2sinθr=2sinθ на симметрию.

    Решение

    Тест на каждый из трех типов симметрии.

    1) Замена (r,θ)(r,θ) на (−r,−θ)(−r,−θ) дает тот же результат. Таким образом, график симметричен относительно прямой θ=π2.θ=π2. −r=2sin(−θ)−r=−2sinθЧетно-нечетная идентичностьr=2sinθУмножить на−1Пройдено−r=2sin(−θ)−r=−2sinθЧетно-нечетная идентичностьr=2sinθУмножить на−1Пройдено
    2) Замена θθ на −θ−θ не дает того же уравнения. Следовательно, график не проходит тест и может быть или не быть симметричным относительно полярной оси. r=2sin(−θ)r=−2sinθЧетно-нечетная идентичностьr=−2sinθ≠2sinθОшибка r=2sin(−θ)r=−2sinθЧетно-нечетная идентичностьr=−2sinθ≠2sinθОшибка
    3) Замена rr на –r–r меняет уравнение и не проходит тест. Граф может быть или не быть симметричным относительно полюса. -r=2sinθ  r=-2sinθ≠2sinθНеудачно-r=2sinθ  r=-2sinθ≠2sinθНеудачно

    Стол 1

    Анализ

    С помощью графического калькулятора мы видим, что уравнение r=2sinθr=2sinθ представляет собой окружность с центром в точке (0,1)(0,1) и радиусом r=1r=1, которая действительно симметрична прямой θ =π2. θ=π2. Мы также можем видеть, что график не симметричен относительно полярной оси или полюса. См. рис. 3.

    Рис. 3

    Попытайся #1

    Проверить уравнение симметрии: r=−2cosθ.r=−2cosθ.

    Графическое построение полярных уравнений по точкам

    Для построения графика в прямоугольной системе координат построим таблицу значений xx и yy. Для построения графика в полярной системе координат построим таблицу значений θθ и rr. Вводим значения θθ в полярное уравнение и вычислить r.r. Однако использование свойств симметрии и нахождение ключевых значений θθ и rr означает, что потребуется меньше вычислений.

    Нахождение нулей и максимумов

    Чтобы найти нули полярного уравнения, мы находим значения θθ, которые дают r=0.r=0. Напомним, что для нахождения нулей полиномиальных функций мы приравниваем уравнение к нулю, а затем находим х.х. Мы используем тот же процесс для полярных уравнений. Установите r=0,r=0 и найдите θ.θ.

    Для многих форм, с которыми мы столкнемся, максимальное значение полярного уравнения находится путем подстановки тех значений θθ в уравнение, которые приводят к максимальному значению тригонометрических функций. Рассмотрим r=5cosθ;r=5cosθ; максимальное расстояние между кривой и полюсом составляет 5 единиц. Максимальное значение функции косинуса равно 1, когда θ=0, θ=0, поэтому наше полярное уравнение имеет вид 5cosθ, 5cosθ, а значение θ=0θ=0 даст максимальное значение |r|.|r|.

    Аналогично, максимальное значение функции синуса равно 1, когда θ=π2,θ=π2, и если наше полярное уравнение имеет вид r=5sinθ,r=5sinθ, значение θ=π2θ=π2 даст максимальное значение |r| .|р|. Мы можем получить дополнительную информацию, рассчитав значения rr при θ=0.θ=0. Эти точки будут точками пересечения полярной оси, которые могут быть полезны при построении графика и определении кривой полярного уравнения.

    Пример 2

    Поиск нулей и максимальных значений для полярного уравнения

    Используя уравнение из примера 1, найдите нули и максимальные значения |r||r| и, при необходимости, полярная ось пересекает r=2sinθ. r=2sinθ.

    Решение

    Чтобы найти нули, приравняйте rr к нулю и найдите θ.θ.

    2sinθ=0sinθ=0θ=sin−10θ=nπ, где n — целое число 2sinθ=0sinθ=0θ=sin−10θ=nπ, где n — целое число

    Подставьте любое из значений θθ в уравнение. Мы будем использовать 0.0.

    r=2sin(0)r=0r=2sin(0)r=0

    Точки (0,0)(0,0) и (0,±nπ)(0,±nπ) являются нулями уравнение. Все они совпадают, поэтому на графике видна только одна точка. Эта точка также является единственным пересечением полярной оси.

    Чтобы найти максимальное значение уравнения, посмотрите на максимальное значение тригонометрической функции sinθ,sinθ, которое возникает, когда θ=π2±2kπθ=π2±2kπ, в результате чего sin(π2)=1.sin(π2)= 1. Замените π2π2 на θ.θ.

    r=2sin(π2)r=2(1)r=2r=2sin(π2)r=2(1)r=2

    Анализ

    Точка (2,π2)(2,π2) будет максимальным значением на графике. Давайте нанесем еще несколько точек, чтобы проверить график окружности. См. Таблицу 2 и Рисунок 4.

    θθ г=2sinθr=2sinθ рр
    0 r=2sin(0)=0r=2sin(0)=0 00
    π6π6 r=2sin(π6)=1r=2sin(π6)=1 11
    π3π3 r=2sin(π3)≈1,73r=2sin(π3)≈1,73 1.731.73
    π2π2 r=2sin(π2)=2r=2sin(π2)=2 22
    2π32π3 r=2sin(2π3)≈1,73r=2sin(2π3)≈1,73 1. 731.73
    5π65π6 r=2sin(5π6)=1r=2sin(5π6)=1 11
    ππ r=2sin(π)=0r=2sin(π)=0 00

    Стол 2

    Рисунок 4

    Попытайся #2

    Не переводя в декартовы координаты, проверить данное уравнение на симметрию и найти нули и максимальные значения |r|:|r|:r=3cosθ.r=3cosθ.

    Исследовательские круги

    Теперь мы рассмотрели уравнение окружности в полярной системе координат. В последних двух примерах одно и то же уравнение использовалось для иллюстрации свойств симметрии и демонстрации того, как находить нули, максимальные значения и точки на графике, по которым строятся графики. Однако круг — это лишь одна из многих фигур в наборе полярных кривых.

    Существует пять классических полярных кривых : кардиоиды , лимаҫоны, лемнискаты, кривые розы и спирали Архимеда . Мы кратко коснемся полярных формул для окружности, прежде чем перейти к классическим кривым и их вариациям.

    Формулы для уравнения окружности

    Некоторые формулы для построения графика окружности в полярных координатах имеют вид r=acosθr=acosθ и r=asinθ,r=asinθ, где aa – диаметр окружности или расстояние от полюса до самого дальнего точка на окружности. Радиус равен |a|2,|a|2 или половине диаметра. Для r=acosθ, r=acosθ,  центр равен (a2,0).(a2,0). Для r=asinθ,r=asinθ центром является (a2,π2).(a2,π2). На рис. 5 показаны графики этих четырех окружностей.

    0. Центр находится в точке (a/2,0). Во-вторых, r=acos(theta), a0. Центр находится в точке (a/2, pi). В-четвертых, это r=asin(theta), a

    Рисунок 5

    Пример 3

    Построение графика полярного уравнения для окружности

    Построение графика r=4cosθ. r=4cosθ.

    Решение

    Во-первых, проверяя уравнение симметрии, мы обнаруживаем, что график симметричен относительно полярной оси. Далее находим нули и максимум |r||r| для r=4cosθ.r=4cosθ. Сначала установите r=0,r=0 и найдите θθ. Таким образом, ноль возникает при θ=π2±kπ.θ=π2±kπ. Ключевой точкой для построения является (0,​π2).(0,​​π2).

    Чтобы найти максимальное значение r,r, обратите внимание, что максимальное значение функции косинуса равно 1, когда θ=0±2kπ.θ=0±2kπ. Подставим θ=0θ=0 в уравнение:

    r=4cosθr=4cos(0)r=4(1)=4r=4cosθr=4cos(0)r=4(1)=4

    Максимальное значение уравнение равно 4. Ключевым моментом для построения графика является (4,0).(4,0).

    Поскольку r=4cosθr=4cosθ симметрично относительно полярной оси, нам нужно вычислить только r -значений для θθ на интервале [0,[0,π].π]. Затем точки в верхнем квадранте могут быть отражены в нижнем квадранте. Составьте таблицу значений, аналогичную таблице 3. График показан на рисунке 6 9.0307 .

    θθ 0 π6π6 π4π4 π3π3 π2π2 2π32π3 3π43π4 5π65π6
    р-р 4 3,46 2,83 2 0 −2 −2,83 −3,46 −4

    Стол 3

    Рисунок 6

    Исследование кардиоид

    Хотя перевод из полярных координат в декартовы координаты в некоторых случаях может показаться более простым, построение классических кривых на самом деле менее сложно в полярной системе. Следующая кривая называется кардиоидой, так как напоминает сердце. Эту форму часто включают в семейство кривых, называемых лимасонами, но здесь мы обсудим кардиоиду отдельно.

    Формулы для кардиоиды

    Формулы для построения графиков кардиоиды имеют вид r=a±bcosθr=a±bcosθ и r=a±bsinθr=a±bsinθ, где a>0,a>0,b>0,b>0, и аб=1.аб=1. Кардиоидный график проходит через полюс, как мы видим на рисунке 7.

    Рисунок 7

    Как

    Имея полярное уравнение кардиоиды, нарисуйте ее график.

    1. Проверьте уравнение для трех типов симметрии.
    2. Найдите нули. Установите r=0.r=0.
    3. Найдите максимальное значение уравнения по максимальному значению тригонометрического выражения.
    4. Составьте таблицу значений rr и θ.θ.
    5. Нанесите точки и нарисуйте график.

    Пример 4

    Начертите график кардиоиды

    Нарисуйте график r=2+2cosθ. r=2+2cosθ.

    Решение

    Во-первых, проверяя уравнение на симметрию, мы обнаруживаем, что график этого уравнения будет симметричен относительно полярной оси. Далее находим нули и максимумы. Полагая r=0,r=0, мы имеем θ=π+2kπ.θ=π+2kπ. Нуль уравнения находится в точке (0,π).(0,π). График проходит через эту точку.

    Максимальное значение r=2+2cosθr=2+2cosθ имеет место, когда cosθcosθ является максимальным, то есть когда cosθ=1cosθ=1 или когда θ=0.θ=0. Подставьте θ=0θ=0 в уравнение и найдите r.r.

    r=2+2cos(0)r=2+2(1)=4r=2+2cos(0)r=2+2(1)=4

    Точка (4,0)(4,0 ) — максимальное значение на графике.

    Мы обнаружили, что полярное уравнение симметрично относительно полярной оси, но, поскольку оно распространяется на все четыре квадранта, нам нужно отобразить значения в интервале [0,π].[0,π]. Затем верхняя часть графика отражается по полярной оси. Далее составляем таблицу значений, как в Таблице 4, а затем наносим точки и рисуем график. См. рис. 8.

    θθ 00 π4π4 π2π2 2π32π3
    р-р 4 3,41 2 1 0

    Стол 4

    Рисунок 8

    Расследование Лимасона

    Слово limaçon на старофранцузском языке означает «улитка» и описывает форму графика. Как упоминалось ранее, кардиоида является членом семейства лимасонов, и мы можем видеть сходство на графиках. Другие изображения в этой категории включают лимасон с одной петлей и лимасон с двумя петлями (или внутренней петлей). Лимасоны с одной петлей иногда называют лимасонами с ямочками, когда 1

    Формулы для лимасонов с одной петлей

    Формулы для построения графика однопетлевого лимасона с ямочками имеют вид r=a±bcosθr=a±bcosθ и r=a±bsinθr=a±bsinθ, где a>0,b>0,a>0, b>0 и 1

    Рисунок 9 Лимасоны с ямочками

    Как

    Имея полярное уравнение для лимасона с одной петлей, нарисуйте график.

    1. Проверьте уравнение на симметрию. Помните, что провал теста на симметрию не означает, что форма не будет демонстрировать симметрию. Часто симметрия может проявляться при нанесении точек.
    2. Найдите нули.
    3. Найдите максимальные значения по тригонометрическому выражению.
    4. Сделать стол.
    5. Нанесите точки и нарисуйте график.

    Пример 5

    Набросок графика однопетлевого лимасона

    Нарисуйте график уравнения r=4−3sinθ.r=4−3sinθ.

    Решение

    Во-первых, проверяя уравнение на симметрию, мы обнаруживаем, что оно не проходит все три теста на симметрию, а это означает, что граф может демонстрировать или не демонстрировать симметрию, поэтому мы не можем использовать симметрию для построения графика. Однако это уравнение имеет график, явно демонстрирующий симметрию относительно прямой θ=π2,θ=π2, но не удовлетворяющий всем трем критериям симметрии. Графический калькулятор сразу же проиллюстрирует отражающую способность графика.

    Затем мы находим нули и максимум и наносим точки отражения для проверки симметрии. Установка r=0r=0 приводит к тому, что θθ не определено. Что это значит? Как θθ может быть неопределенным? Угол θθ не определен для любого значения sinθ>1. sinθ>1. Следовательно, θθ не определено, поскольку не существует значения θθ, для которого sinθ>1.sinθ>1. Следовательно, график не проходит через полюс. Возможно, график пересекает полярную ось, но не на полюсе. Мы можем исследовать другие пересечения, вычислив rr, когда θ=0.θ=0.

    r(0)=4−3sin(0)r=4−3⋅0=4r(0)=4−3sin(0)r=4−3⋅0=4

    Итак, существует хотя бы один пересечение полярной оси в точке (4,0).(4,0).

    Далее, поскольку максимальное значение синуса равно 1 при θ=π2,θ=π2, мы подставим θ=π2θ=π2 в уравнение и решить для r.r. Таким образом, r=1.r=1.

    Составьте таблицу координат, аналогичную таблице 5.

    θθ 00 π6π6 π3π3 π2π2 2π32π3 5π65π6 7π67π6 4π34π3 3π23π2 5π35π3 11π611π6 2π2π
    р-р 4 2,5 1,4 1 1,4 2,5 4 5,5 6,6 7 6,6 5,5 4

    Стол 5

    График показан на рисунке 10.

    Рисунок 10 Лимасон однопетлевой

    Анализ

    Это пример кривой, для которой создание таблицы значений имеет решающее значение для построения точного графика. Тесты на симметрию терпят неудачу; ноль не определен. Хотя может быть очевидным, что уравнение, включающее sinθsinθ, скорее всего, симметрично относительно линии θ=π2,θ=π2, оценка большего количества точек помогает проверить правильность графика.

    Попытайся #3

    Нарисуйте график зависимости r=3−2cosθ.r=3−2cosθ.

    Другой тип лимасона, лимасон с внутренней петлей , назван в честь петли, образованной внутри общей формы лимасона. Он был открыт немецким художником Альбрехтом Дюрером (1471-1528), который раскрыл метод рисования лимасона внутренней петли в своей книге 1525 года Underweysung der Messing . Столетие спустя отец математика Блеза Паскаля, Этьен Паскаль (1588-1651), заново открыл его.

    Формулы для лимасонов внутреннего цикла

    Формулы, порождающие лимасоны внутреннего цикла, имеют вид r=a±bcosθr=a±bcosθ и r=a±bsinθr=a±bsinθ, где a>0,a>0,b>0,b>0, и а<б.а<б. График лимасона внутренней петли проходит через полюс дважды: один раз для внешней петли и один раз для внутренней петли. См. графики на рис. 11.

    Рисунок 11

    Пример 6

    Набросок графика внутреннего цикла Лимасона

    Нарисуйте график r=2+5cosθ.r=2+5cosθ.

    Решение

    Проверяя на симметрию, мы находим, что график уравнения симметричен относительно полярной оси. Затем нахождение нулей показывает, что при r=0,r=0,θ=1,98.θ=1,98. Максимальное |r||r| находится, когда cosθ=1cosθ=1 или когда θ=0,θ=0. Таким образом, максимум находится в точке (7, 0).

    Несмотря на то, что мы нашли симметрию, ноль и максимум, нанесение дополнительных точек поможет определить форму, и тогда появится закономерность.

    См. Таблицу 6.

    θθ 00 π6π6 π3π3 π2π2 2π32π3 5π65π6 7π67π6 4π34π3 3π23π2 5π35π3 11π611π6 2π2π
    рр 7 6,3 4,5 2 −0,5 −2,3 −3 −2,3 −0,5 2 4,5 6,3 7

    Стол 6

    Как и ожидалось, значения начинают повторяться после θ=π. θ=π. График показан на рисунке 12.

    Рисунок 12 Лимасон с внутренней петлей

    Исследование лемнискатов

    Лемниската — это полярная кривая, напоминающая символ бесконечности ∞∞ или цифру 8. Лемниската с центром на полюсе симметрична по определению.

    Формулы для лемнискат

    Формулы, порождающие граф лемнискаты, имеют вид r2=a2cos2θr2=a2cos2θ и r2=a2sin2θr2=a2sin2θ, где a≠0.a≠0. Формула r2=a2sin2θr2=a2sin2θ симметрична относительно полюса. Формула r2=a2cos2θr2=a2cos2θ симметрична относительно полюса, прямой θ=π2,θ=π2 и полярной оси. См. графики на рис. 13.

    Рисунок 13

    Пример 7

    Начертите график лемнискаты

    Нарисуйте график r2=4cos2θ.r2=4cos2θ.

    Решение

    Уравнение обладает симметрией относительно прямой θ=π2,θ=π2, полярной оси и полюса.

    Найдем нули. Сейчас это должно быть обычным делом, но мы подойдем к этому уравнению немного по-другому, сделав замену u=2θ. u=2θ.

    0=4cos2θ0=4cosu0=cosucos−10=π2u=π2Замени 2θ на u.2θ=π2θ=π40=4cos2θ0=4cosu0=cosucos−10=π2u=π2Замени 2θ на u.2θ=π2θ=π4=π2θ0007

    Итак, точка (0,π4)(0,π4) является нулем уравнения.

    Теперь найдем максимальное значение. Поскольку максимум cosu=1cosu=1 при u=0,u=0, максимум cos2θ=1cos2θ=1 при 2θ=0,2θ=0. Таким образом,

    r2=4cos(0)r2=4(1)=4r=±4±2r2=4cos(0)r2=4(1)=4r=±4±2

    Мы имеем максимум при (2 , 0). Поскольку этот график симметричен относительно полюса, линии θ=π2, θ=π2 и полярной оси, нам нужно нанести точки только в первом квадранте.

    Составьте таблицу, аналогичную Таблице 7.

    θθ 0 π6π6 π4π4
    р-р ±2±2 ±2±2 0

    Стол 7

    Нанесите точки на график, как показано на рисунке 14.

    Рисунок 14 Лемниската

    Анализ

    Выполнение замены, такой как u=2θu=2θ, является обычной практикой в ​​математике, поскольку это может упростить вычисления. Однако мы не должны забывать заменить термин замены исходным термином в конце, а затем найти неизвестное.

    Некоторые точки на этом графике могут не отображаться при использовании функции Trace на графическом калькуляторе TI-84, а таблица калькулятора может отображать ошибку для тех же самых точек r.r. Это связано с тем, что для этих значений θ.θ нет реальных квадратных корней. Другими словами, соответствующие r -значений 4cos(2θ)4cos(2θ) являются комплексными числами, потому что под радикалом стоит отрицательное число.

    Исследование кривых розы

    Следующий тип полярного уравнения дает форму лепестка, называемую кривой розы. Хотя графики выглядят сложными, простое полярное уравнение создает закономерность.

    кривые розы

    Формулы, которые создают график кривой розы, задаются как r=acosnθr=acosnθ и r=asinnθr=asinnθ, где a≠0.a≠0. Если nn четно, кривая имеет 2n2n лепестков. Если nn нечетно, кривая имеет nn лепестков. См. рис. 15.

    Рис. 15

    Пример 8

    Эскиз графика кривой розы (
    n Четный)

    Нарисуйте график r=2cos4θ.r=2cos4θ.

    Решение

    Проверяя симметрию, мы снова обнаруживаем, что проверки симметрии не дают полной картины. График симметричен не только относительно полярной оси, но и относительно прямой θ=π2θ=π2 и полюса.

    Теперь найдем нули. Сначала сделайте замену u=4θ.u=4θ.

    0=2cos4θ0=cos4θ0=cosucos−10=uu=π24θ=π2θ=π80=2cos4θ0=cos4θ0=cosucos−10=uu=π24θ=π2θ=π8

    Нуль равен θ=π8.θ=π8. Точка (0,π8)(0,π8) находится на кривой.

    Далее находим максимум |r|.|r|. Мы знаем, что максимальное значение cosu=1cosu=1 при θ=0. θ=0. Таким образом,

    r=2cos(4⋅0)r=2cos(0)r=2(1)=2r=2cos(4⋅0)r=2cos(0)r=2(1)=2

    точка (2,0)(2,0) находится на кривой.

    График кривой розы обладает уникальными свойствами, которые раскрываются в таблице 8.

    θθ 0 π8π8 π4π4 3π83π8 π2π2 5π85π8 3π43π4
    р-р 2 0 −2 0 2 0 −2

    Стол 8

    Так как r=0r=0 при θ=π8,θ=π8, то значения в таблице имеет смысл разделить на π8π8 единиц. Возникает определенная закономерность. Посмотрите на диапазон значений r : 2, 0, −2, 0, 2, 0, −2 и так далее. Это представляет развитие кривой по одному лепестку за раз. Начиная с r=0,r=0, каждый лепесток расширяется на расстояние r=2,r=2, а затем возвращается к нулю 2n2n раз, всего восемь лепестков. См. график на рисунке 16.

    Рисунок 16 Кривая розы, nn даже

    Анализ

    Когда эти кривые построены, лучше всего расположить точки по порядку, как в Таблице 8. Это позволяет нам увидеть, как график достигает максимума (кончик лепестка), петля назад пересекает полюс , достигает противоположного максимума и возвращается к полюсу. Действие продолжается до тех пор, пока не будут нарисованы все лепестки.

    Попытайся #4

    Нарисуйте график зависимости r=4sin(2θ).r=4sin(2θ).

    Пример 9

    Рисование графика кривой розы (
    n Нечетное)

    Нарисуйте график r=2sin(5θ). r=2sin(5θ).

    Решение

    График уравнения показывает симметрию относительно прямой θ=π2.θ=π2. Далее находим нули и максимум. Мы хотим сделать замену u=5θ.u=5θ.

    0=2sin(5θ)0=sinusin−10=0u=05θ=0θ=00=2sin(5θ)0=sinusin−10=0u=05θ=0θ=0

    Максимальное значение вычисляется для угла, где sinθsinθ является максимальным. Следовательно,

    r=2sin(5⋅π2)r=2(1)=2r=2sin(5⋅π2)r=2(1)=2

    Таким образом, максимальное значение полярного уравнения равно 2. Это длина каждого лепестка. Поскольку кривая для нечетного nn дает то же количество лепестков, что и n,n, на графике будет пять лепестков. См. рис. 17.

    Рис. 17 Кривая Розы, nn нечетная

    Создайте таблицу значений, аналогичную Таблице 9.

    θθ 0 π6π6 π3π3 π2π2 2π32π3 5π65π6
    р-р 0 1 −1,73 2 −1,73 1 0

    Стол 9

    Попытайся #5

    Нарисуйте график зависимости r=3cos(3θ). r=3cos(3θ).

    Исследование спирали Архимеда

    Последнее полярное уравнение, которое мы обсудим, — это спираль Архимеда, названная в честь ее первооткрывателя, греческого математика Архимеда (ок. 287 г. до н. э. — ок. 212 г. до н. э.), которому приписывают многочисленные открытия в области геометрии и механики.

    Спираль Архимеда

    Формула, по которой построен график спирали Архимеда, имеет вид r=θr=θ для θ≥0,θ≥0. По мере увеличения θθ rr увеличивается с постоянной скоростью по постоянно расширяющейся, бесконечной, спиралевидной траектории. См. рис. 18.

    Рисунок 18

    Как

    Дана спираль Архимеда на [0,2π],[0,2π], нарисуйте график.

    1. Составьте таблицу значений rr и θθ в данной области.
    2. Нанесите точки и нарисуйте график.

    Пример 10

    Начертите график спирали Архимеда

    Нарисуйте график r=θr=θ на [0,2π]. [0,2π].

    Решение

    Так как rr равно θ,θ, то участок спирали Архимеда начинается на полюсе в точке (0, 0). Хотя график намекает на симметрию, формальной симметрии в отношении прохождения тестов на симметрию нет. Кроме того, максимальное значение отсутствует, если домен не ограничен.

    Создайте таблицу, например Таблицу 10.

    θθ π4π4 π2π2 3π23π2 7π47π4 2π2π
    р-р 0,785 1,57 3,14 4,71 5,50 6,28

    Стол 10

    Обратите внимание, что значения r представляют собой просто десятичную форму угла, измеренного в радианах. Мы можем увидеть их на графике на рисунке 19.

    Рисунок 19 Спираль Архимеда

    Анализ

    Область определения этой полярной кривой [0,2π].[0,2π]. Однако в общем случае область определения этой функции равна (−∞,∞).(−∞,∞). Начертить уравнение спирали Архимеда довольно просто, хотя на изображении кажется, что оно сложное.

    Попытайся #6

    Нарисуйте график зависимости r=-θr=-θ на интервале [0,4π].[0,4π].

    Сводка кривых

    В этом разделе мы исследовали ряд кажущихся сложными полярных кривых. Рисунок 20 и Рисунок 21 суммируют графики и уравнения для каждой из этих кривых.

    0, б>0, а/б=1. (C) — лимасоны с одной петлей. r= a + или — bcos(тета), или r= a + или — bsin(тета). а>0, б>0, 10, б>0, а

    Рисунок 20

    =0.»>

    Рисунок 21

    8.4 Секционные упражнения

    Устно

    1.

    Опишите три типа симметрии в полярных графах и сравните их с симметрией декартовой плоскости.

    2.

    Какие из трех типов симметрий для полярных графов соответствуют симметриям относительно оси x , оси y и начала координат?

    3.

    Какие шаги необходимо выполнить при построении графика полярных уравнений?

    4.

    Описать формы графиков кардиоид, лимасонов и лемнискат.

    5.

    Какая часть уравнения определяет форму графика полярного уравнения?

    Графический

    В следующих упражнениях проверьте уравнение симметрии.

    6.

    r=5cos3θr=5cos3θ

    7.

    r=3−3cosθr=3−3cosθ

    8.

    r=3+2sinθr=3+2sinθ

    9.

    r=3sin2θr=3sin2θ

    10.

    г=4р=4

    11.

    r=2θr=2θ

    12.

    r=4cosθ2r=4cosθ2

    13.

    r=2θr=2θ

    14.

    r=31-cos2θr=31-cos2θ

    15.

    r=5sin2θr=5sin2θ

    Для следующих упражнений нарисуйте полярное уравнение. Определите название фигуры.

    16.

    r=3cosθr=3cosθ

    17.

    r=4sinθr=4sinθ

    18.

    r=2+2cosθr=2+2cosθ

    19.

    r=2−2cosθr=2−2cosθ

    20.

    r=5−5sinθr=5−5sinθ

    21.

    r=3+3sinθr=3+3sinθ

    22.

    r=3+2sinθr=3+2sinθ

    23.

    r=7+4sinθr=7+4sinθ

    24.

    r=4+3cosθr=4+3cosθ

    25.

    r=5+4cosθr=5+4cosθ

    26.

    r=10+9cosθr=10+9cosθ

    27.

    r=1+3sinθr=1+3sinθ

    28.

    r=2+5sinθr=2+5sinθ

    29.

    r=5+7sinθr=5+7sinθ

    30.

    r=2+4cosθr=2+4cosθ

    31.

    r=5+6cosθr=5+6cosθ

    32.

    r2=36cos(2θ)r2=36cos(2θ)

    33.

    r2=10cos(2θ)r2=10cos(2θ)

    34.

    r2=4sin(2θ)r2=4sin(2θ)

    35.

    r2=10sin(2θ)r2=10sin(2θ)

    36.

    r=3sin(2θ)r=3sin(2θ)

    37.

    r=3cos(2θ)r=3cos(2θ)

    38.

    r=5sin(3θ)r=5sin(3θ)

    39.

    r=4sin(4θ)r=4sin(4θ)

    40.

    r=4sin(5θ)r=4sin(5θ)

    41.

    r=-θr=-θ

    42.

    r=2θr=2θ

    43.

    r=-3θr=-3θ

    Технология

    В следующих упражнениях используйте графический калькулятор, чтобы начертить график полярного уравнения.

    44.

    r=1θr=1θ

    45.

    r=1θr=1θ

    46.

    r=2sinθtanθ,r=2sinθtanθ, циссоида

    47.

    r=21−sin2θr=21−sin2θ, гиппопед

    48.

    r=5+cos(4θ)r=5+cos(4θ)

    49.

    r=2-sin(2θ)r=2-sin(2θ)

    50.

    r=θ2r=θ2

    51.

    г=θ+1r=θ+1

    52.

    r=θsinθr=θsinθ

    53.

    r=θcosθr=θcosθ

    В следующих упражнениях используйте графическую утилиту для построения графика каждой пары полярных уравнений в области [0,4π][0,4π], а затем объясните различия, показанные на графиках.

    54.

    r=θ,r=−θr=θ,r=−θ

    55.

    r=θ,r=θ+sinθr=θ,r=θ+sinθ

    56.

    r=sinθ+θ,r=sinθ−θr=sinθ+θ,r=sinθ−θ

    57.

    r=2sin(θ2),r=θsin(θ2)r=2sin(θ2),r=θsin(θ2)

    58.

    r=sin(cos(3θ))r=sin(3θ)r=sin(cos(3θ))r=sin(3θ)

    59.

    В графической утилите постройте график r=sin(165θ)r=sin(165θ) на [0[0, 4π]4π], [0[0, 8π]8π], [0[0, 12π]12π] и [0[0, 16π].16π]. Опишите эффект увеличения ширины домена.

    60.

    В графической утилите постройте график и эскиз r=sinθ+(sin(52θ))3r=sinθ+(sin(52θ))3 на [0,4π].[0,4π].

    61.

    С помощью графической утилиты нарисуйте каждое полярное уравнение. Объясните сходства и различия, которые вы видите на графиках.

    r1=3sin(3θ)r2=2sin(3θ)r3=sin(3θ)r1=3sin(3θ)r2=2sin(3θ)r3=sin(3θ)

    62.

    С помощью графической утилиты нарисуйте каждое полярное уравнение. Объясните сходства и различия, которые вы видите на графиках.

    r1=3+3cosθr2=2+2cosθr3=1+cosθr1=3+3cosθr2=2+2cosθr3=1+cosθ

    63.

    С помощью графической утилиты нарисуйте каждое полярное уравнение. Объясните сходства и различия, которые вы видите на графиках.

    r1=3θr2=2θr3=θr1=3θr2=2θr3=θ

    Расширения

    В следующих упражнениях начертите каждое полярное уравнение на одном и том же наборе полярных осей и найдите точки пересечения.

    64.

    r1=3+2sinθ,r2=2r1=3+2sinθ,r2=2

    65.

    r1=6−4cosθ,r2=4r1=6−4cosθ,r2=4

    66.

    r1=1+sinθ,r2=3sinθr1=1+sinθ,r2=3sinθ

    67.

    r1=1+cosθ,r2=3cosθr1=1+cosθ,r2=3cosθ

    68.

    r1=cos(2θ),r2=sin(2θ)r1=cos(2θ),r2=sin(2θ)

    69.

    r1=sin2(2θ)r1=sin2(2θ), r2=1−cos(4θ)r2=1−cos(4θ)

    70.

    r1=3,r2=2sin(θ)r1=3,r2=2sin(θ)

    71.

    r12=sinθ,r22=cosθr12=sinθ,r22=cosθ

    72.

    r1=1+cosθr1=1+cosθ, r2=1−sinθr2=1−sinθ

    полярный график полярное уравнение полярная кривая розы 3 листа розы симметричны относительно оси y

    • Кто мы
    • Бесплатные видео
    • Лучшие учителя
    • охваченных тем
    • Членство
    • О
    • Математика
    • Наука
    • Английский
    • Подготовка к тесту
    • Колледж
    • Войти
    • Начните бесплатный пробный период
  • Все
  • 80 Предварительный расчет
  • 62 Алгебра
  • 55 Алгебра 2
  • 17 Химия
  • 16 Тригонометрия
  • 13 Пре-алгебра
  • 11 Расчет
  • 11 Консультации в колледже
  • 10 AP Исчисление AB
  • 8 Геометрия
    • Семейства полярных кривых: розы
      Предварительный расчет Полярные координаты и комплексные числа

      Как описать Розы, семейство кривых с уравнениями r=acos(b*theta) или r=asin(b*theta), когда b >=2 и является целым числом.

      полярный графикполярное уравнениеполярная криваярозасимметрична относительно оси xсимметрична относительно оси y.

    • Графики полярных уравнений
      Предварительный расчет Полярные координаты и комплексные числа

      Как нарисовать простую полярную кривую, нанеся точки.

      полярный графикполярное уравнение

    • Симметрия полярных графиков
      Предварительный расчет Полярные координаты и комплексные числа

      Как определить, симметричен ли график полярного уравнения относительно оси X.

      полярный графикполярное уравнениесимметрияотражениеось xсимметрично относительно оси x

    • Полярность
      Химия Химические связи

      Как электроотрицательность влияет на полярность молекул.

      полярностьэлектроотрицательностьполярные молекулы

    • Семейства полярных кривых: конические сечения
      Предварительный расчет Полярные координаты и комплексные числа

      Как описывать полярные уравнения конических сечений.

      полярный графикполярное уравнениеполярная криваяконические сеченияфокусисходныйполюсполюспараболасимметричный относительно оси xвершина параболы

    • строк в полярных координатах
      Предварительный расчет Полярные координаты и комплексные числа

      Как найти полярное уравнение прямой, проходящей через начало координат (или полюс).

      полярные координатыlineoriginполярное уравнениепрямоугольное уравнение

    • Формула расстояния в полярных координатах
      Предварительный расчет Полярные координаты и комплексные числа

      Как вывести формулу расстояния для полярных координат.

      полярные координатызакон косинусовтреугольники

    • Введение в полярные координаты
      Предварительный расчет Полярные координаты и комплексные числа

      Как описать положение точки с помощью полярных координат и как преобразовать полярные координаты в прямоугольные.

      полярная оськоординаты начала полюсапрямоугольные координатыполярные координатысистема координаттеорема Пифагорасинекосинустангенс

    • Преобразование полярных координат в прямоугольные
      Предварительный расчет Полярные координаты и комплексные числа

      Как преобразовать полярное уравнение в прямоугольную форму.

      полярные координатыпрямоугольные координатыполярные уравнениялинииокружностипифагоровы теоремысинекосинустангенс

    • Семейства полярных кривых: круги, кардиоды и лимакон
      Предварительный расчет Полярные координаты и комплексные числа

      Как построить график особых случаев семейства r = a + b cos (тета), когда a или b = 0.

      полярный графикполярное уравнениеполярная криваякругицентр радиусзаполнение квадрата

    • Преобразование прямоугольных координат в полярные
      Предварительный расчет Полярные координаты и комплексные числа

      Как преобразовать прямоугольные координаты точки в полярные координаты.

      полярные координатыпрямоугольные координатытеорема Пифагорасинекосинустангенс

    • Решение систем уравнений графическим способом
      Алгебра Решение систем уравнений

      Как решить систему уравнений с помощью графика.

      система уравнений решение системы уравнений построение графика

    • Графики квадратных уравнений
      Алгебра Квадратные уравнения и функции

      Как нарисовать квадратное уравнение вручную.

      x перехваты перехватывершинаось симметриидискриминантзаменить

    • Вершина и ось симметрии
      Алгебра Квадратные уравнения и функции

      Как найти вершину и ось симметрии параболы.

      вершина оси симметрии

    • Отражение y = f(-x)
      Предварительный расчет Введение в функции

      Как отразить график y = f(x) по оси y.

      функцииродительские функцииотражение

    • Соединение графиков, таблиц и уравнений
      Алгебра Графики и функции

      Как связать линейные уравнения, таблицы значений и их графики.

      наклонный перехватчик

    • Нахождение уравнения нарисованной линии
      Алгебра Линейные уравнения и их графики

      Как написать уравнение нарисованной линии.

      точка наклона форма пересечения наклона формы пересечения наклона

    • Симметрия графов: нечетные и четные функции
      Предварительный расчет Введение в функции

      Как распознать график четной или нечетной функции.

      четные функции нечетные функции симметричные относительно оси у симметричные относительно начала координат родительские функции

    • Графические линии с использованием перехватов
      Алгебра Линейные уравнения и их графики

      Как построить линию с помощью точек пересечения x и y.

      x перехват стандартная форма перехвата

    • Параметрические уравнения и движение
      Предварительный расчет Векторы и параметрические уравнения

      Как построить график параметрической кривой и как исключить параметр, чтобы получить прямоугольное уравнение для кривой.

      время положенияпараметрические уравненияуравнение прямоугольной траекторииустранение параметровфункция квадратного корнянаправление движения

    Вопросы? Обратная связь? работает на программном обеспечении для живого чата Olark

    Полярные координаты: Графики | Алгебра и тригонометрия

    Планеты движутся в пространстве по эллиптическим периодическим орбитам вокруг Солнца, как показано на (Рисунок). {2}\,[/latex]описывает отношение между [латексом]\,x\,[/ латекс] и [латекс]\,у\,[/латекс] на декартовой сетке, a полярное уравнение описывает взаимосвязь между [латексом]\,r\,[/латексом] и [латексом]\,\тета \,[/латекс] на полярной сетке. Напомним, что пара координат [латекс]\,\влево(r,\тета \вправо)\,[/латекс] указывает, что мы двигаемся против часовой стрелки от полярной оси (положительные x -оси) на угол [латекс] \,\theta ,\,[/latex]и продолжить луч от полюса (начала)[латекс]\,r\,[/латекс]единиц в направлении [латекс]\,\тета .\,[/ латекс]Все точки, удовлетворяющие полярному уравнению, находятся на графике.

    Симметрия — это свойство, которое помогает нам распознавать и строить график любого уравнения. Если уравнение имеет график, симметричный относительно оси, это означает, что если мы сложим график пополам по этой оси, часть графика с одной стороны совпадет с частью с другой стороны. Выполнив три теста, мы увидим, как применять свойства симметрии к полярным уравнениям. Далее мы будем использовать симметрию (в дополнение к построению ключевых точек, нулей и максимумов [латекса]\,r)\,[/латекса] для построения графика полярного уравнения.

    В первом тесте мы рассматриваем симметрию относительно линии [латекс]\,\theta =\frac{\pi }{2}\,[/латекс]( y -оси). Мы заменяем [латекс]\,\left(r,\theta\right)\,[/latex] на [латекс]\,\left(-r,-\theta \right)\,[/latex], чтобы определить, новое уравнение эквивалентно исходному уравнению. Например, предположим, что нам дано уравнение [латекс]\,r=2\mathrm{sin}\,\theta ;[/latex]

    [латекс]\begin{array}{ll}\,\,\, \,r=2\mathrm{sin}\,\theta\hfill & \hfill \\ -r=2\mathrm{sin}\left(-\theta\right)\begin{array}{cccc}& & & \end{массив}\hfill & \text{Заменить}\,\left(r,\theta\right)\,\text{с}\left(-r,-\theta\right).\hfill \\ — r=-2\mathrm{sin}\,\theta\hfill & \text{Идентификация:}\mathrm{sin}\left(-\theta\right)=-\mathrm{sin}\,\theta .\hfill \\ \,\,\,\,\,r=2\mathrm{sin}\,\theta \hfill & \text{Умножить обе части на}-1. \hfill \end{массив}[/latex]

    Это уравнение проявляет симметрию относительно прямой [латекс]\,\тета =\фракция {\pi }{2}.[/латекс]

    Во втором тесте мы рассматриваем симметрию относительно полярной оси ( [латекс]\,x[/латекс]-ось). Мы заменяем [латекс]\,\left(r,\theta \right)\,[/latex] на [латекс]\,\left(r,-\theta \right)\,[/latex]или [латекс] \,\left(-r,\pi -\theta \right)\,[/latex] для определения эквивалентности между тестируемым уравнением и исходным. Например, предположим, что нам дано уравнение[латекс]\,r=1-2\mathrm{cos}\,\theta .[/латекс]

    [латекс]\begin{array}{ll}r=1-2\mathrm{cos}\,\theta \hfill & \hfill \\ r=1-2\mathrm{cos}\left(-\theta \right)\begin{array}{cccc}& & & \end{array}\hfill & \text{Replace}\left(r,\theta \right)\,\text{with}\,\left(r ,-\theta \right).\hfill \\ r=1-2\mathrm{cos}\,\theta \hfill & \text{четная/нечетная идентичность}\hfill \end{массив}[/latex]

    График этого уравнения проявляет симметрию относительно полярной оси.

    В третьем тесте мы рассматриваем симметрию относительно полюса (начала). Мы заменяем [латекс]\,\left(r,\theta \right)\,[/latex] на [латекс]\,\left(-r,\theta \right)\,[/latex], чтобы определить, проверенное уравнение эквивалентно исходному уравнению. Например, предположим, что нам дано уравнение [латекс]\,r=2\mathrm{sin}\left(3\theta \right).[/latex]

    [латекс]\begin{array}{c}\,\,\,\,\,r=2\mathrm{sin}\left(3\theta\right)\\ -r=2\mathrm{sin }\left(3\theta \right)\end{array}[/latex]

    Уравнение не прошло тест на симметрию, но это не означает, что оно несимметрично относительно полюса. Прохождение одного или нескольких тестов на симметрию подтверждает, что симметрия будет отображаться на графике. Однако непрохождение тестов на симметрию не обязательно означает, что график не будет симметричным относительно линии [латекс]\,\тета =\фракция{\пи} {2},\,[/латекс] полярной оси или столб. В этих случаях мы можем подтвердить существование симметрии, нанеся отражающие точки на кажущуюся ось симметрии или полюс. Проверка на симметрию — это метод, который упрощает построение графиков полярных уравнений, но его применение несовершенно.

    Проверка симметрии

    Полярное уравнение описывает кривую на полярной сетке. График полярного уравнения может быть оценен для трех типов симметрии, как показано на (Рисунок).

    Рис. 2. (a) График симметричен относительно прямой [латекс]\,\theta =\frac{\pi }{2}\,[/латекс](ось Y) при замене[ латекс]\,\left(r,\theta \right)\,[/latex]с [латексом]\,\left(-r,-\theta \right)\,[/latex] дает эквивалентное уравнение. (b) Граф симметричен относительно полярной оси (оси x), если заменить [латекс]\,\left(r,\theta \right)\,[/latex]на [латекс]\,\left( r,-\theta\right)\,[/latex]или[latex]\,\left(-r,\mathrm{\pi -}\theta\right)\,[/latex] дает эквивалентное уравнение. (c) Граф симметричен относительно полюса (начала координат), если заменить [латекс]\,\left(r,\theta\right)\,[/latex] на [латекс]\,\left(-r, \theta \right)\,[/latex] дает эквивалентное уравнение.

    Как

    Имея полярное уравнение, проверьте его симметрию.

    1. Подставьте соответствующую комбинацию компонентов вместо [латекс]\,\left(r,\theta \right):[/latex][латекс]\,\left(-r,-\theta \right)\, [/latex]for[латекс]\,\theta =\frac{\pi }{2}\,[/latex]симметрия;[латекс]\,\left(r,-\theta \right)\,[/ латекс] для симметрии полярной оси; и [латекс]\,\влево(-r,\тета\вправо)\,[/латекс]для симметрии относительно полюса.
    2. Если результирующие уравнения эквивалентны в одном или нескольких тестах, график обеспечивает ожидаемую симметрию.

    Проверка полярного уравнения на симметрию

    Проверка уравнения[латекс]\,r=2\mathrm{sin}\,\theta \,[/латекс] на симметрию.

    Показать решение

    Анализ

    С помощью графического калькулятора мы видим, что уравнение[латекс]\,r=2\mathrm{sin}\,\theta \,[/латекс] представляет собой окружность с центром в[латекс]\,\ left(0,1\right)\,[/latex]с радиусом[latex]\,r=1\,[/latex] и действительно симметричен прямой[latex]\,\theta =\frac{\pi {2}.\,[/latex]Мы также можем видеть, что график не симметричен относительно полярной оси или полюса. См. (Рисунок).

    Рис. 3.

    Попробуйте

    Проверьте уравнение симметрии: [латекс]\,r=-2\mathrm{cos}\,\theta .[/латекс]

    Показать решение

    Построение графика полярных уравнений по точкам

    Для построения графика в прямоугольной системе координат мы строим таблицу значений [латекс]\,х\,[/латекс] и [латекс]\,у\,[/латекс]. Чтобы построить график в полярной системе координат, мы строим таблицу значений [латекс]\,\тета\,[/латекс] и [латекс]\,r\,[/латекс]. Мы вводим значения[латекс]\,\тета\,[/латекс] в полярное уравнение и вычисляем[латекс]\,р.\,[/латекс]Однако, используя свойства симметрии и находя ключевые значения[латекс ]\,\theta \,[/latex] и [latex]\,r\,[/latex] означают, что потребуется меньше вычислений.

    Нахождение нулей и максимумов

    Чтобы найти нули полярного уравнения, мы решаем значения [латекс]\,\тета\,[/латекс], что дает [латекс]\,r=0.\, [/latex] Напомним, что для нахождения нулей полиномиальных функций мы устанавливаем уравнение равным нулю, а затем решаем для[latex]\,x.\,[/latex]. Мы используем тот же процесс для полярных уравнений. Установите[latex]\,r=0,\,[/latex]и решите для[latex]\,\theta .[/latex]

    Для многих форм, с которыми мы столкнемся, максимальное значение полярного уравнения равно можно найти, подставив те значения [латекс]\,\тета \,[/латекс] в уравнение, которые приводят к максимальному значению тригонометрических функций. Рассмотрим [латекс]\,r=5\mathrm{cos}\,\theta ;\,[/латекс]максимальное расстояние между кривой и полюсом составляет 5 единиц. Максимальное значение функции косинуса равно 1, когда [латекс]\,\тета = 0,\,[/латекс], поэтому наше полярное уравнение имеет вид [латекс]\,5\mathrm{cos}\,\тета,\,[ /latex] и значение[latex]\,\theta =0\,[/latex] даст максимальное значение[latex]\,|r|.[/latex]

    Аналогично, максимальное значение функции синуса равно 1, когда [латекс]\,\тета =\фракция{\пи }{2},\,[/латекс] и если наше полярное уравнение равно [латекс]\,r =5\mathrm{sin}\,\theta ,\,[/latex]значение [латекс]\,\theta =\frac{\pi }{2}\,[/latex] даст максимум[латекс] \,|r|.\,[/latex] Мы можем найти дополнительную информацию, вычислив значения [latex]\,r\,[/latex], когда [latex]\,\theta =0.\,[/latex] Эти точки будут точками пересечения полярной оси, которые могут быть полезны при построении графика и определении кривой полярного уравнения.

    Нахождение нулей и максимальных значений для полярного уравнения

    Используя уравнение на (рис. ), найдите нули и максимальные значения[latex]\,|r|\,[/latex]и, при необходимости, точки пересечения полярной оси [латекс]\,r=2\mathrm{sin}\,\theta .[/латекс]

    Показать решение

    Анализ

    Точка[latex]\,\left(2,\frac{\pi }{2}\right)\,[/latex] будет максимальным значением на графике. Давайте нанесем еще несколько точек, чтобы проверить график окружности. См. (Рисунок) и (Рисунок).

    [латекс]\тета [/латекс] [латекс]r=2\mathrm{sin}\,\тета [/латекс] [латекс]г[/латекс]
    0 [латекс]r=2\mathrm{sin}\left(0\right)=0[/latex] [латекс]0[/латекс]
    [латекс]\frac{\pi }{6}[/латекс] [латекс] r = 2 \ mathrm {sin} \ left (\ frac {\ pi {6} \ right) = 1 [/latex] [латекс]1[/латекс]
    [латекс]\frac{\pi }{3}[/латекс] [латекс] r = 2 \ mathrm {sin} \ left (\ frac {\ pi} {3} \ right) \ приблизительно 1,73 [/ латекс] [латекс]1,73[/латекс]
    [латекс]\frac{\pi }{2}[/латекс] [латекс] r = 2 \ mathrm {sin} \ left (\ frac {\ pi} {2} \ right) = 2 [/latex] [латекс]2[/латекс]
    [латекс]\frac{2\pi }{3}[/латекс] [латекс]r=2\mathrm{sin}\left(\frac{2\pi }{3}\right)\приблизительно 1,73[/latex] [латекс]1,73[/латекс]
    [латекс]\frac{5\pi }{6}[/латекс] [латекс]r=2\mathrm{sin}\left(\frac{5\pi }{6}\right)=1[/latex] [латекс]1[/латекс]
    [латекс]\pi [/латекс] [латекс]r=2\mathrm{sin}\left(\pi \right)=0[/latex] [латекс]0[/латекс]

    Рис. 4.

    Попробуйте

    Без преобразования в декартовы координаты проверьте данное уравнение на симметрию и найдите нули и максимальные значения [latex]\,|r|:\,[/latex] [латекс]\,r=3\mathrm{cos}\,\theta .[/латекс]

    Показать решение

    Исследовательские кружки

    Теперь мы рассмотрели уравнение окружности в полярной системе координат. В последних двух примерах одно и то же уравнение использовалось для иллюстрации свойств симметрии и демонстрации того, как находить нули, максимальные значения и точки на графике, по которым строятся графики. Однако круг — это лишь одна из многих фигур в наборе полярных кривых.

    Существует пять классических полярных кривых : кардиоиды , лимаҫоны, лемнискаты, розы и спирали Архимеда . Мы кратко коснемся полярных формул для окружности, прежде чем перейти к классическим кривым и их вариациям.

    Формулы для уравнения окружности

    Некоторые формулы для построения графика окружности в полярных координатах имеют вид[latex]\,r=a\mathrm{cos}\,\theta \,[/latex ]and[латекс]\,r=a\mathrm{sin}\,\theta ,[/latex], где [латекс]\,a\,[/latex] — диаметр окружности или расстояние от полюса до самая дальняя точка окружности. Радиус равен [латекс]\,\фрак{|а|}{2},[/латекс] или половине диаметра. Для [латекс]\,r=a\mathrm{cos}\,\theta , [/latex] центр [латекс]\,\left(\frac{a}{2},0\right).\, [/latex]Для [латекса]\,r=a\mathrm{sin}\,\theta ,[/latex] центр [латекс]\,\left(\frac{a}{2},\frac{ \pi }{2}\right).\,[/latex](Рисунок) показывает графики этих четырех окружностей.

    Рисунок 5.

    Набросок графика полярного уравнения для окружности

    Нарисуйте график [латекс]\,r=4\mathrm{cos}\,\theta .[/латекс]

    Показать Решение

    Изучение кардиоид

    Хотя перевод полярных координат в декартовы координаты в некоторых случаях может показаться более простым, построение классических кривых в полярной системе на самом деле менее сложно. Следующая кривая называется кардиоидой, так как напоминает сердце. Эту форму часто включают в семейство кривых, называемых лимасонами, но здесь мы обсудим кардиоиду отдельно.

    Формулы для кардиоиды

    Формулы, по которым строятся графики кардиоиды, задаются как [латекс]\,r=a±b\mathrm{cos}\,\theta \,[/latex]и[латекс]\ ,r=a±b\mathrm{sin}\,\theta\,[/latex],где[латекс]\,a>0,\,\,b>0,\,[/latex]и[латекс]\ ,\frac{a}{b}=1. \,[/latex]Кардиоидный график проходит через полюс, как мы видим на (рис.).

    Рис. 7.

    Как сделать

    Имея полярное уравнение кардиоиды, нарисуйте ее график.

    1. Проверьте уравнение для трех типов симметрии.
    2. Найдите нули. Установить[латекс]\,r=0.[/латекс]
    3. Найдите максимальное значение уравнения по максимальному значению тригонометрического выражения.
    4. Создайте таблицу значений для [латекс]\,r\,[/латекс] и [латекс]\,\тета .[/латекс]
    5. Нанесите точки и нарисуйте график.

    Набросок графика кардиоиды

    Набросок графика [латекс]\,r=2+2\mathrm{cos}\,\theta .[/латекс]

    Показать решение

    Изучение Лимасона

    Слово limaçon на старофранцузском языке означает «улитка» и описывает форму графика. Как упоминалось ранее, кардиоида является членом семейства лимасонов, и мы можем видеть сходство на графиках. Другие изображения в этой категории включают лимасон с одной петлей и лимасон с двумя петлями (или внутренней петлей). Лимасоны с одной петлей иногда называют лимасонами с ямочками, когда [латекс]\,1<\frac{a}{b}<2\,[/latex], и выпуклыми лимасонами, когда[латекс]\,\frac{a }{b}\ge 2.\,[/latex]

    Формулы для лимасона с одной петлей

    Формулы для построения графика однопетлевого лимасона с ямочками задаются как [латекс]\,r=a±b\mathrm{cos}\,\theta \,[/latex ] и [латекс]\,r=a±b\mathrm{sin}\,\theta \,[/latex], где [латекс]\,a>0,\,b>0,\,\,\text{ и 1<}\frac{a}{b}<2.\,[/latex]Все четыре графика показаны на (рис.).

    Рисунок 9. Лимасон с ямочками

    Как сделать

    Имея полярное уравнение для лимасона с одной петлей, нарисуйте график.

    1. Проверьте уравнение на симметрию. Помните, что провал теста на симметрию не означает, что форма не будет демонстрировать симметрию. Часто симметрия может проявляться при нанесении точек.
    2. Найдите нули.
    3. Найдите максимальные значения по тригонометрическому выражению.
    4. Сделать стол.
    5. Нанесите точки и нарисуйте график.

    Набросок графика лимасона с одной петлей

    Нарисуйте уравнение[latex]\,r=4-3\mathrm{sin}\,\theta .[/latex]

    Показать решение

    Анализ

    Это пример кривой, для которой составление таблицы значений имеет решающее значение для построения точного графика. Тесты на симметрию терпят неудачу; ноль не определен. Хотя может быть очевидным, что уравнение, включающее [латекс]\,\mathrm{sin}\,\theta \,[/latex], скорее всего, симметрично относительно линии [латекс]\,\theta =\frac{\pi {2},[/latex] оценка большего количества баллов помогает проверить правильность графика.

    Попробуйте

    Нарисуйте график [латекс]\,r=3-2\mathrm{cos}\,\theta .[/латекс]

    Показать решение

    Другой тип лимасона, лимасон с внутренней петлей , назван в честь петли, образованной внутри общей формы лимасона. Он был открыт немецким художником Альбрехтом Дюрером (1471-1528), который раскрыл метод рисования лимасона внутренней петли в своей книге 1525 года Underweysung der Messing . Столетие спустя отец математика Блеза Паскаля, Этьен Паскаль (1588-1651), заново открыл его.

    Формулы для лимасонов внутреннего контура

    Формулы, порождающие лимасоны внутреннего цикла, задаются как [латекс]\,r=a±b\mathrm{cos}\,\theta\,[/latex]и[латекс]\,r=a±b\ mathrm{sin}\,\theta\,[/latex], где [латекс]\,a>0,\,\,b>0,\,[/latex] и [латекс]\,\,a

    Рисунок 11.

    Набросок графика внутреннего цикла Limaçon

    Набросок графика [latex]\,r=2+5\text{cos}\,\theta .[/latex] 9{2}=4\mathrm{cos}\,2\theta .[/latex]

    Показать решение

    Анализ

    Подстановка, такая как [латекс]\,u=2\тета \,[/латекс], является обычной практикой в ​​математике, поскольку она может упростить вычисления. Однако мы не должны забывать заменить термин замены исходным термином в конце, а затем найти неизвестное.

    Некоторые точки на этом графике могут не отображаться при использовании функции Trace на графическом калькуляторе TI-84, а таблица калькулятора может отображать ошибку для этих же точек [latex]\,r.\,[/latex ]Это потому, что для этих значений [латекс]\,\тета .\,[/латекс] нет реальных квадратных корней.\,[/латекс] Другими словами, соответствующие r -значения [латекс]\,\sqrt{4\mathrm{cos}\left(2\theta \right)}\,[/latex]
    являются комплексными числами, потому что под радикалом стоит отрицательное число.

    Исследование кривых розы

    Следующий тип полярного уравнения дает форму лепестка, называемую кривой розы. Хотя графики выглядят сложными, простое полярное уравнение создает закономерность.

    Кривые розы

    Формулы, которые создают график кривой розы, задаются как [латекс]\,r=a\mathrm{cos}\,n\theta \,[/latex] и [латекс]\,r =a\mathrm{sin}\,n\theta \,[/latex],где[latex]\,a\ne 0. \,[/latex]Если[latex]\,n\,[/latex]четно , кривая имеет[латекс]\,2n\,[/латекс]лепестки. Если [латекс]\,n\,[/латекс]нечетный, кривая имеет[латекс]\,n\,[/латекс]лепестки. См. (Рисунок).

    Рисунок 15.

    Набросок графика кривой розы (

    n Четный)

    Нарисуйте график [латекс]\,r=2\mathrm{cos}\,4\theta .[/latex ]

    Показать решение

    Анализ

    При построении этих кривых точки лучше располагать по порядку, как показано на (Рисунок). Это позволяет нам увидеть, как график достигает максимума (кончик лепестка), зацикливается на пересечении полюса, достигает противоположного максимума и возвращается к полюсу. Действие продолжается до тех пор, пока не будут нарисованы все лепестки.

    Попробуйте

    Нарисуйте график [латекс]\,r=4\mathrm{sin}\left(2\theta \right).[/latex]

    Показать решение

    Набросок графика кривой розы (

    n Нечетное)

    Набросок графика [латекс]\,r=2\mathrm{sin}\left(5\theta \right). [/latex]

    Показать решение

    Попробуйте

    Нарисуйте график [латекс]r=3\mathrm{cos}\left(3\theta \right).[/latex]

    Показать решение

    Исследование спирали Архимеда

    Последнее полярное уравнение, которое мы обсудим, — это спираль Архимеда, названная в честь ее первооткрывателя, греческого математика Архимеда (ок. 287 г. до н. э. — ок. 212 г. до н. э.), которому приписывают многочисленные открытия в области геометрии и механики.

    Спираль Архимеда

    Формула, по которой построен график спирали Архимеда, имеет вид [латекс]\,r=\theta \,[/latex]
    for[latex]\,\theta \ge 0.\ ,[/latex]По мере того, как[латекс]\,\тета\,[/латекс]увеличивается,[латекс]\,r\,[/латекс]
    увеличивается с постоянной скоростью в постоянно расширяющейся, бесконечной, спиральной дорожка. См. (Рисунок).

    Рисунок 18.

    Как сделать

    Дана спираль Архимеда над [латексом]\,\слева[0,2\пи \справа],[/латекс] нарисуйте график.

    1. Составьте таблицу значений для [латекс]\,r\,[/латекс] и [латекс]\,\тета \,[/латекс] для заданного домена.
    2. Нанесите точки и нарисуйте график.

    Набросок графика спирали Архимеда

    Нарисуйте график [латекс]\,r=\theta \,[/latex]над [латексом]\,\left[0,2\pi \right].[ /латекс]

    Показать решение

    Анализ

    Область определения этой полярной кривой [латекс]\,\влево[0,2\пи \право].\,[/латекс]Однако в общем случае область определения этой функции [латекс]\ ,\left(-\infty ,\infty \right).\,[/latex]Нарисовать уравнение спирали Архимеда довольно просто, хотя на изображении кажется, что оно сложное.

    Попробуйте

    Нарисуйте график [латекс]\,r=-\тета \,[/латекс] на интервале [латекс]\,\слева[0,4\пи \справа].[/латекс]

    Показать решение

    Сводка кривых

    Рис. 20.

    В этом разделе мы рассмотрели ряд кажущихся сложными полярных кривых. (Рисунок) и (Рисунок) суммируют графики и уравнения для каждой из этих кривых.

    Рис. 21.

    Доступ к этим онлайн-ресурсам для получения дополнительных инструкций и практики с графиками полярных координат.

    • Графики полярных уравнений, часть 1
    • Графики полярных уравнений Часть 2
    • Анимация: графики полярных уравнений
    • Графики полярных уравнений на TI-84

    Ключевые понятия

    • Начертить полярные уравнения проще, если мы можем проверить уравнения на симметрию относительно линии[латекс]\,\тета =\фракция{\пи} {2},\,[/латекс ] полярная ось или полюс.
    • Существует три теста на симметрию, которые показывают, будет ли график полярного уравнения демонстрировать симметрию. Если уравнение не проходит тест на симметрию, график может демонстрировать или не демонстрировать симметрию. См. (Рисунок).
    • Полярные уравнения можно представить в виде графика, составив таблицу значений для [латекс]\,\тета\,[/латекс] и [латекс]\,r. [/латекс]
    • Максимальное значение полярного уравнения находится путем подстановки значения [латекс]\,\тета \,[/латекс], которое приводит к максимальному значению тригонометрического выражения.
    • Нули полярного уравнения можно найти, установив [латекс]\,r=0\,[/латекс] и найдя [латекс]\,\тета .\,[/латекс] См. (рисунок).
    • Некоторые формулы, которые создают график окружности в полярных координатах, задаются как [латекс]\,r=a\mathrm{cos}\,\theta \,[/latex]и [латекс]\,r=a\mathrm {sin}\,\theta .\,[/latex] См. (рисунок).
    • Формулы, по которым строятся графики кардиоиды, задаются как [латекс]\,r=a±b\mathrm{cos}\,\theta \,[/latex]и[латекс]\,r=a±b\ mathrm{sin}\,\theta ,\,[/latex]для[латекса]\,a>0,\,\,b>0,\,[/latex]и[латекс]\,\frac{a} {b}=1.\,[/latex] См. (Рисунок).
    • Формулы, по которым строятся графики лимасона с одной петлей, задаются как [латекс]\,r=a±b\mathrm{cos}\,\theta \,[/latex]и[латекс]\,r=a ±b\mathrm{sin}\,\theta \,[/latex]for[latex]\,1<\frac{a}{b}<2. \,[/latex]См. (рисунок).
    • 9{2}\mathrm{sin}\,2\theta ,\,[/latex],где[latex]\,a\ne 0.[/latex]См. (рисунок).
    • Формулы, по которым строятся графики розовых кривых, задаются как [латекс]\,r=a\mathrm{cos}\,n\theta \,[/latex]и[латекс]\,r=a\mathrm{sin }\,n\theta ,\,[/latex]где[латекс]\,a\ne 0;\,[/latex]если [латекс]\,n\,[/latex]четно, существуют[латекс ]\,2n\,[/latex]лепестков, а если [латекс]\,n\,[/латекс]нечётно, то [латекс]\,n\,[/латекс]лепестков. См. (Рисунок) и (Рисунок).
    • Формула, по которой строится график спирали Архимеда, имеет вид[latex]\,r=\theta ,\,\,\theta \ge 0.\,[/latex]См. (рисунок).

    Упражнения по разделам

    Вербальные

    Опишите три типа симметрии в полярных графах и сравните их с симметрией декартовой плоскости.

    Показать решение

    Какие из трех типов симметрии полярных графов соответствуют симметрии относительно оси x , оси y и начала координат?

    Какие шаги необходимо выполнить при построении графика полярных уравнений?

    Показать решение

    Описать формы графиков кардиоид, лимасонов и лемнискат.

    Какая часть уравнения определяет форму графика полярного уравнения?

    Показать решение

    Графический

    В следующих упражнениях проверьте уравнение симметрии.

    [латекс]r=5\mathrm{cos}\,3\theta [/latex]

    [латекс]r=3-3\mathrm{cos}\,\theta [/latex]

    Показать решение

    [латекс]r=3+2\mathrm{sin}\,\theta [/latex]

    [латекс]r=3\mathrm{sin}\,2\theta [/latex]

    Показать решение

    [латекс]r=4[/латекс] 9{2}\theta }[/latex]

    [latex]r=\sqrt{5\mathrm{sin}\,2\theta }[/latex]

    Показать решение

    Для следующих упражнений нарисуйте полярное уравнение. Определите название фигуры.

    [латекс]r=3\mathrm{cos}\,\theta [/latex]

    [латекс]r=4\mathrm{sin}\,\theta [/latex]

    Показать решение

    [латекс]r=2+2\mathrm{cos}\,\theta [/latex]

    [латекс]r=2-2\mathrm{cos}\,\theta [/latex]

    Показать решение

    [латекс]r=5-5\mathrm{sin}\,\тета [/латекс]

    [латекс]r=3+3\mathrm{sin}\,\theta [/латекс]

    Показать решение

    [латекс]r=3+2\mathrm{sin}\,\theta [/latex]

    [латекс]r=7+4\mathrm{sin}\,\theta [/latex]

    Показать решение

    [латекс]r=4+3\mathrm{cos}\,\theta [/latex]

    [латекс]r=5+4\mathrm{cos}\,\theta [/latex]

    Показать решение

    [латекс]r=10+9\mathrm{cos}\,\theta [/latex]

    [латекс]r=1+3\mathrm{sin}\,\theta [/latex]

    Показать решение

    9{2}=10\mathrm{sin}\left(2\theta \right)[/latex]

    Показать решение

    [латекс]r=3\текст{sin}\left(2\theta\right)[/latex]

    [латекс]r=3\text{cos}\left(2\theta\right)[/ латекс]

    Показать решение

    [латекс]r=5\текст{sin}\left(3\theta\right)[/latex]

    [латекс]r=4\text{sin}\left(4\theta\right)[/ латекс]

    Показать решение

    [латекс]r=4\текст{sin}\left(5\theta \right)[/latex]

    [латекс]r=-\theta [/latex]

    Показать решение

    [латекс]r=2\тета [/латекс]

    [латекс]r=-3\тета [/латекс]

    Показать решение

    Технология

    В следующих упражнениях используйте графический калькулятор, чтобы нарисовать график полярного уравнения. {2}\theta }[/latex], гиппопед 9{2}[/latex]

    [latex]r=\theta +1[/latex]

    Показать решение

    [латекс]r=\theta \mathrm{sin}\,\theta [/latex]

    [latex]r=\theta \mathrm{cos}\,\theta [/latex]

    Показать решение

    В следующих упражнениях используйте графическую утилиту для построения графика каждой пары полярных уравнений в области [латекс]\,\слева[0,4\пи \справа]\,[/латекс], а затем объясните показанные различия на графиках.

    [латекс]r=\theta ,r=-\theta [/latex]

    [латекс]r=\theta ,r=\theta +\mathrm{sin}\,\theta [/latex]

    Показать решение

    [латекс]r=\mathrm{sin}\,\theta +\theta ,r=\mathrm{sin}\,\theta -\theta [/latex]

    [латекс]r=2\mathrm{sin }\left(\frac{\theta}{2}\right),r=\theta\mathrm{sin}\left(\frac{\theta}{2}\right)[/latex]

    Показать решение

    [латекс]r=\mathrm{sin}\left(\mathrm{cos}\left(3\theta\right)\right)\,\,r=\mathrm{sin}\left(3\theta\ right)[/latex]

    В графической утилите graph[latex]\,r=\mathrm{sin}\left(\frac{16}{5}\theta \right)\,[/latex]on[ латекс]\,\влево[0,4\pi \справа],\влево[0,8\pi \вправо],\влево[0,12\pi \вправо],\,[/латекс]и[латекс] \,\left[0,16\pi \right]. \,[/latex]Опишите эффект увеличения ширины домена. 9{3}\,[/latex]on[latex]\,\left[0,4\pi \right].[/latex]

    С помощью графической утилиты нарисуйте каждое полярное уравнение. Объясните сходства и различия, которые вы видите на графиках.

    [латекс]\begin{array}{l}\begin{array}{l}\\ {r}_{1}=3\mathrm{sin}\left(3\theta\right)\end{array }\hfill \\ {r}_{2}=2\mathrm{sin}\left(3\theta\right)\hfill \\ {r}_{3}=\mathrm{sin}\left(3\ theta \right)\hfill \end{array}[/latex]

    Показать решение

    С помощью графической утилиты нарисуйте каждое полярное уравнение. Объясните сходства и различия, которые вы видите на графиках.

    [латекс]\begin{array}{l}\begin{array}{l}\\ {r}_{1}=3+3\mathrm{cos}\,\theta \end{array}\hfill \\ {r}_{2}=2+2\mathrm{cos}\,\theta \hfill \\ {r}_{3}=1+\mathrm{cos}\,\theta \hfill \end{ array}[/latex]

    В графической утилите нарисуйте график каждого полярного уравнения. Объясните сходства и различия, которые вы видите на графиках.

    [латекс]\begin{array}{l}\begin{array}{l}\\ {r}_{1}=3\theta \end{array}\hfill \\ {r}_{2} =2\theta \hfill \\ {r}_{3}=\theta \hfill \end{массив}[/latex]

    Показать решение

    Extensions

    В следующих упражнениях начертите каждое полярное уравнение на одном и том же наборе полярных осей и найдите точки пересечения.

    [латекс]{r}_{1}=3+2\mathrm{sin}\,\theta,\,{r}_{2}=2[/латекс]

    [латекс]{r}_ {1}=6-4\mathrm{cos}\,\theta ,\,{r}_{2}=4[/latex]

    Показать решение

    [латекс] {r}_{1}=1+\mathrm{sin}\,\theta ,\,{r}_{2}=3\mathrm{sin}\,\theta [/latex]

    [латекс]{r}_{1}=1+\mathrm{cos}\,\theta,\,{r}_{2}=3\mathrm{cos}\,\theta [/latex] 9{2}=\mathrm{cos}\,\theta [/latex]

    Показать решение

    [латекс] {r}_{1}=1+\mathrm{cos}\,\theta ,\,{r}_{2}=1-\mathrm{sin}\,\theta [/latex]

    Глоссарий

    Спираль Архимеда
    полярная кривая, заданная как [латекс]\,r=\theta . \,[/latex]При умножении на константу уравнение выглядит как[латекс]\,r=a\тета .\,[/латекс]Как [latex]\,r=\theta ,\,[/latex]кривая продолжает расширяться по спирали над доменом.
    кардиоидная
    г. член семейства кривых лимасон, названный в честь сходства с сердцем; его уравнение задается как [латекс]\,r=a±b\mathrm{cos}\,\theta \,[/latex] and[latex]\,r=a±b\mathrm{sin}\,\theta ,\,[/латекс], где[латекс]\,\фракция{а}{б}=1[/латекс]
    выпуклый изгиб
    тип лимасона с одной петлей, представленный [латекс]\,r=a±b\mathrm{cos}\,\theta\,[/latex]и[латекс]\,r=a±b\mathrm{sin }\,\theta \,[/latex]такие, что[latex]\,\frac{a}{b}\ge 2[/latex]
    лиман с ямочками
    г.
    тип лимасона с одной петлей, представленный [латекс]\,r=a±b\mathrm{cos}\,\theta\,[/latex] и [латекс]\,r=a±b\mathrm{sin }\,\theta \,[/latex] такой, что [latex]\,1<\frac{a}{b}<2[/latex]
    лимасон с внутренней петлей
    полярная кривая, похожая на кардиоиду, но с внутренней петлей; проходит через шест дважды; представлено [латекс]\,r=a±b\mathrm{cos}\,\theta\,[/latex] и[latex]\text{ }r=a±b\mathrm{sin}\,\theta \ текст{ }[/латекс]где[латекс]\,а
    одноконтурный лиман
    полярная кривая, представленная [латекс]\,r=a±b\mathrm{cos}\,\theta\,[/latex] и [латекс]\,r=a±b\mathrm{sin}\,\ theta \,[/latex] такой, что [latex]a>0,b>0,[/latex]и[latex]\,\frac{a}{b}>1;[/latex] может быть ямчатым или выпуклым ; не проходит через столб
    полярное уравнение
    уравнение, описывающее кривую на полярной сетке.

    Добавить комментарий

    Ваш адрес email не будет опубликован. Обязательные поля помечены *